Tagnawa BSWR (Biochemistry) Flashcards

1
Q

A newly married African American couple, both from families having histories of good health, are about to have a child. If the incidence of the sickle cell trait is approximately 1 1⁄2 among persons of African descenta in the United States, what is the chance that they will give birth to a child that is affected by sickle cell disease?
a. 1/12
b. 1/24
c. 1/96
d. 1/288
e. 1/576

A

e. 1/576

How well did you know this?
1
Not at all
2
3
4
5
Perfectly
2
Q

A 25-year-old African-American male with sickle cell anemia, who has been hospitalized several times from painful sickle cell crises, has successfully been free of these crises since he has been on hydroxyurea therapy. Treatment with hydroxyurea results in which of the following?
a. An increase in the oxygen affinity of HbS
b. An increase in the levels of hemoglobin F (HbF) in red blood cells
c. A decease cooperatively in oxygen binding by HbS
d. A posttranslational modification of HbS that prevents polymerization
e. A decreased ability of HbS to bind 2,3-biphosphoglycerate (2,3-BPG)

A

b. An increase in the levels of hemoglobin F (HbF) in red blood cells

How well did you know this?
1
Not at all
2
3
4
5
Perfectly
3
Q

A pregnant woman is able to transfer oxygen to her fetus because fetal hemoglobin has a greater affinity for oxygen than does adult hemoglobin. Why is the affinity of fetal hemoglobin for oxygen higher?
a. The tense form of hemoglobin is more prevalent in the circulation of the fetus
b. There is less 2,3-BPG in the fetal circulation as compared to maternal circulation
c. Fetal hemoglobin binds 2,3-BPG with fewer ionic bond than the adult form
d. The Bohr effect is enhanced in the fetus
e. The oxygen-binding curve of fetal hemoglobin is shifted to the right

A

c. Fetal hemoglobin binds 2,3-BPG with fewer ionic bond than the adult form

How well did you know this?
1
Not at all
2
3
4
5
Perfectly
4
Q

Influenza virus is a class Vb virus, which means that it has a single (–)- stranded RNA for its genome. Which of the following best describes the immediate fate of this (–)-RNA when the virus enters the host cell?

a. It is used directly to encode viral proteins.
b. It is used as a template to synthesize a (+)-strand viral messenger RNA (mRNA).
c. It is used as a template to synthesize viral DNA.
d. It is converted to a provirus.
e. It is integrated into the host cell genome.

A

b. It is used as a template to synthesize a (+)-strand viral messenger RNA (mRNA).

How well did you know this?
1
Not at all
2
3
4
5
Perfectly
5
Q

If a double-stranded DNA molecule undergoes two rounds of replication in an in vitro system that contains all of the necessary enzymes and nucleoside triphosphates that have been labeled with 32P, which of the following best describes the distribution of radioactivity in the four resulting DNA molecules?

a. Exactly one of the molecules contains no radioactivity.
b. Exactly one of the molecules contains radioactivity in only one strand.
c. Two of the molecules contain radioactivity in both strands.
d. Three of the molecules contain radioactivity in both strands.
e. All four molecules contain radioactivity in only one strand.

A

c. Two of the molecules contain radioactivity in both strands.

How well did you know this?
1
Not at all
2
3
4
5
Perfectly
6
Q

A 48-year-old man has had a lengthy history of skin cancer. In the past 6 years he has had over 30 neoplasms removed from sun-exposed areas and has been diagnosed with xeroderma pigmentosum. Which of the following best describes the enzymatic defect in patients with xeroderma pigmentosum?
a. DNA polymerase α
b. DNA polymerase γ
c. DNA ligase
d. Excision repair enzymes
e. RNA polymerase III

A

d. Excision repair enzymes

How well did you know this?
1
Not at all
2
3
4
5
Perfectly
7
Q

A 44-year-old woman who recently lost her job because of absenteeism, presents to her physician complaining of loss of appetite, fatigue, muscle weakness, and emotional depression. The physical examination reveals a somewhat enlarged liver that feels firm and nodular, and there is a hint of jaundice in the sclerae and a hint of alcohol on her breath. The initial laboratory profile included a hematological analysis that showed that she had an anemia with enlarged red blood cells (macrocytic). A bone marrow aspirate confirmed the suspicion that she has a megaloblastic anemia because it showed a greater than normal number of red and white blood cell precursors, most of which were larger than normal. Further analyses revealed that her serum folic acid level was 2.9 ng/mL (normal = 6 to 15), her serum B12 level was 153 pg/mL (normal = 150 to 750), and her serum iron level was normal. The patient’s megaloblastic anemia is most likely caused by which of the following?
a. A decreased synthesis of methionine
b. A decreased conversion of dUMP to dTMP
c. A decrease in the synthesis of phosphatidyl choline
d. A decrease in the levels of succinyl CoA
e. A decreased synthesis of dUTP

A

b. A decreased conversion of dUMP to dTMP

How well did you know this?
1
Not at all
2
3
4
5
Perfectly
8
Q

A patient presents with a urinary tract infection and is prescribed a combination drug containing trimethoprim and sulfamethoxazole. These drugs are effective because they do which of the following?
a. Bind to operons to prevent synthesis of bacterial mRNA
b. Block transport across bacterial cell walls
c. Inhibit bacterial synthesis of cobalamin (B12)
d. Inhibit bacterial synthesis of THF
e. Inhibit synthesis of phospholipids in bacteria

A

d. Inhibit bacterial synthesis of THF

How well did you know this?
1
Not at all
2
3
4
5
Perfectly
9
Q

Methotrexate is often used as a chemotherapeutic agent to treat patients with leukemia. This drug is effective because it inhibits cells in which part of the cell cycle?
a. G1 phase
b. S phase
c. M phase
d. G2 phase
e. G0 phase

A

b. S phase

How well did you know this?
1
Not at all
2
3
4
5
Perfectly
10
Q

Leukemia patients are often given the compound Leucovorin (N5-formyl THF) following treatment with the drug methotrexate. Why is leucovorin useful as part of this treatment protocol?
a. It facilitates the uptake of methotrexate by cells
b. It can be converted to THF by bypassing DHFR
c. It acts as an activator of thymidylate synthase
d. It prevents the uptake of methotrexate by normal cells
e. It stimulates cells of the immune system

A

b. It can be converted to THF by bypassing DHFR

How well did you know this?
1
Not at all
2
3
4
5
Perfectly
11
Q

Because of the close interrelationship between the vitamins, patients with deficiencies of either folate or vitamin B12 exhibit similar symptoms. Which of the following tests would best help distinguish between a folate and vitamin B12 deficiency?
a. Activity of methionine synthase
b. Blood level of cystathionine
c. Blood level of homocysteine
d. Blood level of methionine
e. Blood level of methylmalonate

A

e. Blood level of methylmalonate

How well did you know this?
1
Not at all
2
3
4
5
Perfectly
12
Q

A muscular 25-year-old male presents with dermatitis and an inflamed tongue. A history reveals that he has been consuming raw eggs as part of his training regimen for the past 6 months.
a. Ascorbic acid (vitamin C)
b. Biotin
c. Cobalamin (vitamin B12)
d. Folic acid
e. Niacin (vitamin B3 )
f. Pantothenic acid
g. Riboflavin (vitamin B2 )
h. Thiamine (vitamin B1 )

A

b. Biotin

How well did you know this?
1
Not at all
2
3
4
5
Perfectly
13
Q

A 30-year-old male goes to his dentist complaining of loosening teeth. Examination also reveals his gums are swollen, purple, and spongy. The dentist also notes that the patient’s fingers have multiple splinter hemorrhages near the distal ends of the nail and that a wound on the patient’s forearm has failed to heal properly.
a. Ascorbic acid (vitamin C)
b. Biotin
c. Cobalamin (vitamin B12)
d. Folic acid
e. Niacin (vitamin B3 )
f. Pantothenic acid
g. Riboflavin (vitamin B2 )
h. Thiamine (vitamin B1 )

A

a. Ascorbic acid (vitamin C)

How well did you know this?
1
Not at all
2
3
4
5
Perfectly
14
Q

A female neonate is found to have a small spina bifida in her lower spinal column that could affect bladder and lower limb function.
a. Ascorbic acid (vitamin C)
b. Biotin
c. Cobalamin (vitamin B12)
d. Folic acid
e. Niacin (vitamin B3 )
f. Pantothenic acid
g. Riboflavin (vitamin B2 )
h. Thiamine (vitamin B1 )

A

d. Folic acid

How well did you know this?
1
Not at all
2
3
4
5
Perfectly
15
Q

The current therapeutic strategy for patients who have been infected with HIV is a multidrug regimen known as highly active antiretroviral therapy (HAART). One type of drug used in this therapy is a nucleoside/ nucleotide analog, such as didanosine. Which of the following best describes the mechanism of action of these drugs?
a. They inhibit the synthesis of viral proteins.
b. They directly bind to and inhibit reverse transcriptase.
c. They prevent the hydrolysis of the viral polyprotein.
d. They prematurely terminate the DNA synthesized by reverse transcriptase
e. They inhibit the viral enzyme integrase

A

d. They prematurely terminate the DNA synthesized by reverse transcriptase

How well did you know this?
1
Not at all
2
3
4
5
Perfectly
16
Q

Given the mRNA nucleotide sequence, choose the best protein sequence that will likely result. (Use the amino acid table in Questions [13.2] and [13.3] in Case 13, keeping in mind that [T] and [U] are analogous).mRNA5ʹ′AUCGGAUGUCUCGGGUUCUGUAAAGGUAA UC 3ʹ′
a. Met-Ser-Arg-Val-Leu
b. Ser-Arg-Val-Leu
c. Met-Leu-Ser-Val
d. Ser-Arg-Val-Phe-Phe e. Pro-Ser-Val-Gly

A

a. Met-Ser-Arg-Val-Leu

How well did you know this?
1
Not at all
2
3
4
5
Perfectly
17
Q

One of the steps in the PCR amplification of DNA fragments is the denaturation step in which the temperature is raised to break the hydrogen bonds that make up the base pairing. Which of the following DNA fragments would most likely require the greatest increase in temperature to cause complete denaturation?
a. 5ʹ′-C-A-A-T-G-T-A-A-T-T-G-C-A-T-3ʹ′
3ʹ′-G-T-T- A-C-A-T-T-A-A-C-G-T-A-5ʹ′
b. 5ʹ′-A-T-A-T-A-T-A-T-A-T-A-T-A-T-3ʹ′
3ʹ′-T- A-T- A-T-A-T-A-T-A-T-A-T-A-5ʹ′
c. 5ʹ′-A-A-C-C-G-G-A-C-C-G-C-G-A-T-3”
3ʹ′-T- T-G-G-C-C-T-G-G-C-G-C-T-A-5ʹ′
d. 5ʹ′-A-G-A-G-A-G-A-G-A-G-A-G-A-G-3ʹ′
3ʹ′-T-C-T-C-T-C-T-C-T- C-T-C-T-C-5ʹ′
e. 5ʹ′-G-A-C-T-G-T-A-A-T-A-C-G-A-T-3ʹ′
3ʹ′-C-T-G-A-C-A-T-T-A-T-G-C-T-A-5ʹ′

A

c. 5ʹ′-A-A-C-C-G-G-A-C-C-G-C-G-A-T-3”
3ʹ′-T- T-G-G-C-C-T-G-G-C-G-C-T-A-5ʹ′

How well did you know this?
1
Not at all
2
3
4
5
Perfectly
18
Q

Restriction enzymes are used to cleave genomic DNA into smaller fragments. Which of the following single-strand DNA sequences has the best potential to be a site of action for a restriction endonuclease?
a. T–A–G–C–T–T
b. C–T–G–C–A–G
c. A–A–C–C–A–A
d. G–T–G–T–G–T
e. A–A–A–C–C–C

A

b. C–T–G–C–A–G

How well did you know this?
1
Not at all
2
3
4
5
Perfectly
19
Q

A 21-year-old woman was abducted when she went to the local convenience store. Her body was found the next morning in a wooded area behind the store. The autopsy revealed that she had been sexually assaulted and strangled. Crime scene investigators were able to collect a semen sample from vaginal fluid as well as tissue samples from underneath the victim’s fingernails. DNA samples were obtained from three suspects besides the victim. A variable number of tandem repeats (VNTR) analysis was performed on the DNA samples from the evidence collected, the victim, and the suspect, and the results were compared. Which of the following techniques is the most appropriately applied for this analysis?
a. Allele-specific oligonucleotide probes
b. DNA sequencing
c. Northern blot
d. Southern blot
e. Western blot

A

d. Southern blot

How well did you know this?
1
Not at all
2
3
4
5
Perfectly
20
Q

The thyroid hormones T3 and T4 are synthesized in the follicular cells of the thyroid gland. From which of the following essential amino acids are the thyroid hormones synthesized?
a. Isoleucine
b. Lysine
c. Methionine
d. Phenylalanine
e. Valine

A

d. Phenylalanine

How well did you know this?
1
Not at all
2
3
4
5
Perfectly
21
Q

The thyroid hormones T3 and T4 bind to the thyroid hormone receptor (THR) in the target cells. Which of the following mechanisms best describes the role of the THR?
a. It activates adenylyl cyclase to produce cAMP.
b. It activates the phosphoinositide cascade.
c. It is a soluble guanylyl cyclase.
d. It is a tyrosine kinase.
e. It is a transcription factor.

A

e. It is a transcription factor.

How well did you know this?
1
Not at all
2
3
4
5
Perfectly
22
Q

A 26-year-old male presents complaining of heat intolerance, heavy sweating, tremulousness, and feeling “jittery inside.” Physical examination reveals reddened conjunctiva and warm and moist palms, but the thyroid gland was not visibly enlarged. Which of the following tests would be most helpful to obtain an accurate diagnosis?
a. Electrocardiogram
b. Free thyroxine level
c. Serum cortisol levels
d. Serum electrolytes
e. Serum glucose level

A

b. Free thyroxine level

How well did you know this?
1
Not at all
2
3
4
5
Perfectly
23
Q

A 30-month-old female child whose growth rate has been in the lower 10th percentile over the last year presents with chronic, nonproductive cough and diarrhea with foul-smelling stools. She is diagnosed as having cystic fibrosis. For which of the following vitamins is this child most likely to be at risk of deficiency?
A. Ascorbic acid (vitamin C)
B. Biotin
C. Folic acid
D. Retinol (vitamin A)
E. Riboflavin (vitamin B2 )

A

D. Retinol (vitamin A)

How well did you know this?
1
Not at all
2
3
4
5
Perfectly
24
Q

Some forms of genetic diseases such as cystic fibrosis and sickle cell anemia can be diagnosed by detecting restriction fragment length polymorphisms (RFLPs). Which of the following is most likely to be used in an RFLP analysis?
a. Dideoxynucleotides
b. Mass spectrometry
c. Northern blot
d. Southern blot
e. Western blot

A

d. Southern blot

How well did you know this?
1
Not at all
2
3
4
5
Perfectly
25
Q

Your patient has been diagnosed with cystic fibrosis and has been determined to have the most common mutation, the ΔF508 gene. Which of the following is the most cost- and time- effective method for testing family members to see who are carriers of the mutation?
a. Allele-specific oligonucleotide probe analysis
b. DNA fingerprinting analysis
c. DNA sequencingz
d. Restriction length fragment polymorphism analysis

A

a. Allele-specific oligonucleotide probe analysis

How well did you know this?
1
Not at all
2
3
4
5
Perfectly
26
Q

Transfer of the peptide from the peptidyl tRNA to the aminoacyl-tRNA and formation of a peptide bond.
a. Aminoglycosides
b. Chloramphenicol
c. Erythromycin
d. Gentamicin
e. Kanamycin
f. Lincosamides
g. Neomycin
h. Puromycin
i. Streptomycin
j. Tetracycline

A

B. Chlorampenicol

How well did you know this?
1
Not at all
2
3
4
5
Perfectly
27
Q

Binding of aminoacyl-tRNA in the A-site of the ribosomal complex
a. Aminoglycosides
b. Chloramphenicol
c. Erythromycin
d. Gentamicin
e. Kanamycin
f. Lincosamides
g. Neomycin
h. Puromycin
i. Streptomycin
j. Tetracycline

A

j. Tetracycline

How well did you know this?
1
Not at all
2
3
4
5
Perfectly
28
Q

Translocation of the peptidyl tRNA from the A-site to the P-site.
a. Aminoglycosides
b. Chloramphenicol
c. Erythromycin
d. Gentamicin
e. Kanamycin
f. Lincosamides
g. Neomycin
h. Puromycin
i. Streptomycin
j. Tetracycline

A

c. Erythromycin

How well did you know this?
1
Not at all
2
3
4
5
Perfectly
29
Q

The 6-year-old son of a migrant worker is brought to a clinic with chills, headache, nausea, vomiting, and sore throat. The examining physician notes a persistent grayish colored membrane near the tonsils. History reveals that the patient has not been immunized against diphtheria. Diphtheria toxin is potentially lethal in this unimmunized patient because it causes which of the following?
a. Inactivates an elongation factor required for translocation in protein synthesis
b. Binds to the ribosome and prevents peptide bond formation
c. Prevents binding of mRNA to the 60S ribosomal subunit
d. Inactivates an initiation factor
e. Inhibits the synthesis of aminoacyl-charged tRNA

A

a. Inactivates an elongation factor required for translocation in protein synthesis

How well did you know this?
1
Not at all
2
3
4
5
Perfectly
30
Q

Replication of a particular DNA sequence is noted to be under inhibitory control usually. However, when substance “A” is added, it binds to a repressor, rendering the repressor inactive and allowing transcription to occur. Which of the following terms describes agent “A”?
a. Histone
b. Operon
c. Polymerase
d. Transcriber
e. Inducer

A

e. Inducer

How well did you know this?
1
Not at all
2
3
4
5
Perfectly
31
Q

A 38-year-old woman, who works as an administrative assistant for a large company, opened a package and found a suspicious white powder. Analysis of the powder indicates that it contained traces of the bacterium Bacillus anthracis. The woman was treated with ciprofloxacin, an effective antibiotic. Ciprofloxacin’s mechanism of action is best described as an inhibition of which of the following?
A. Bacterial dihydrofolate reductase
B. Bacterial peptidyl transferase activity
C. Bacterial RNA polymerase
D. DNA gyrase
E. DNA polymerase III

A

D. DNA gyrase

How well did you know this?
1
Not at all
2
3
4
5
Perfectly
32
Q

The Rubenstein-Taybi syndrome (RTS) is a genetic disease that is characterized by distinctive facial features, broad thumbs, broad big toes, and mental retardation. The affected gene is CBP (CREB-binding protein gene), which codes for a transcriptional activator. The RTS phenotype is best expressed by a haploinsufficiency model, in which two functional copies of the gene are required to produce sufficient CBP for proper development. CBP has a histone acetyltransferase activity, which does which of the following?
A. Inhibits RNA polymerase II
B. Helps expose the promoters of genes
C. Inhibits the splicing of heterogeneous nuclear RNA (hnRNA) to messenger RNA (mRNA)
D. Prevents the addition of a poly-A tail to mRNA
E. Activates the formation of nucleosomes

A

B. Helps expose the promoters of genes

How well did you know this?
1
Not at all
2
3
4
5
Perfectly
33
Q

Acetylation and deacetylation of lysine residues on histone proteins provide one mechanism by which transcription can be activated or repressed. Which one of the histone proteins is least likely to participate in this process?
a. H1
b. H2A
c. H2B
d. H3
e. H4

A

a. H1

How well did you know this?
1
Not at all
2
3
4
5
Perfectly
34
Q

Hereditary retinoblastoma is a genetic disease that is inherited as an autosomal dominant trait. Patients with hereditary retinoblastoma develop tumors of the retina early in life, usually in both eyes. The affected gene (RB1) was the first tumor suppressor gene to be identified. Which of the following best describes the function of the protein encoded by the RB1 gene?
a. It binds transcription factors required for expression of DNA replication enzymes
b. It allosterically inhibits DNA polymerase.
c. It binds to the promoter region of DNA and prevents transcription.
d. It phosphorylates signal-transduction proteins

A

a. It binds transcription factors required for expression of DNA replication enzymes

How well did you know this?
1
Not at all
2
3
4
5
Perfectly
35
Q

Mutations in the tumor suppressor gene BRCA1 are transmitted in an autosomal dominant fashion. When a cell is transformed to a tumor cell in individuals who have inherited one mutant allele of this tumor suppressor gene, which of the following most likely occurs?
a. A transcription factor is over expressed.
b. Deletion or mutation of the normal gene on the other chromosome
c. Chromosomal translocation.
d. Gene duplication of the mutant gene

A

b. Deletion or mutation of the normal gene on the other chromosome

How well did you know this?
1
Not at all
2
3
4
5
Perfectly
36
Q

Women who inherit one mutant BRCA1 gene have a 60 percent chance of developing breast cancer by the age of 50. The protein produced by the BRCA1 gene has been found to be involved in the repair of DNA double-strand breaks. Which of the following processes is most likely to be adversely affected by a deficiency in the BRCA1 protein?
a. Removal of thymine dimers
b. Removal of RNA primers
c. Removal of carcinogen adducts
d. Homologous recombination
e. Correction of mismatch errors

A

d. Homologous recombination

How well did you know this?
1
Not at all
2
3
4
5
Perfectly
37
Q

Micawley Talltwin is a 7-year-old child star who is brought to his pediatrician by his parents after they noticed that he felt very fatigued. They also noted that his abdomen seemed to be enlarged. Examination reveals an enlarged spleen. Further history reveals that he has been taking vitamins and iron supplements over the last few months. Laboratory tests show a microcytic anemia and elevated iron levels in tissues. Which of the following conditions is most consistent with the findings in this patient?
a. Aplastic anemia
b. Cooley anemia
c. Pernicious anemia
d. Thalassemia major
e. Thalassemia minor

A

e. Thalassemia minor

How well did you know this?
1
Not at all
2
3
4
5
Perfectly
38
Q

Micawley Talltwin is a 7-year-old child star who is brought to his pediatrician by his parents after they noticed that he felt very fatigued. They also noted that his abdomen seemed to be enlarged. Examination reveals an enlarged spleen. Further history reveals that he has been taking vitamins and iron supplements over the last few months. Laboratory tests show a microcytic anemia and elevated iron levels in tissues.

After diagnosing Micawley Talltwin and ascertaining he is not agranulocytic, his physician prescribes subcutaneous infusion of deferroxamine, an iron chelator, and monitors him for several weeks. What is his condition most likely to be on reexamination?
a. Iron levels decrease, but he remains anemic.
b. Iron levels decrease, and anemia recovers.
c. He develops irreversible and severe agranulocytosis
d. Iron and vitamin C levels decrease.

A

a. Iron levels decrease, but he remains anemic.

How well did you know this?
1
Not at all
2
3
4
5
Perfectly
39
Q

An electrophoretic analysis of Micawley Talltwin’s hemoglobin indicates that although there is a decrease in the relative amount of the β-chain with respect to the α-chain, both the β- and the α-chains migrate at the same position as normal chains. Most likely his anemia is caused by which of the following?
a. A defect in an enzyme involved in heme synthesis
b. A point mutation in the coding region of the gene coding for the β-chain
c. A frameshift mutation in the coding region of the gene coding for the β-chain
d. A mutation in the promoter of the β-chain gene
e. A mutation in the structural gene of the β-chain

A

d. A mutation in the promoter of the β-chain gene

How well did you know this?
1
Not at all
2
3
4
5
Perfectly
40
Q

As a medical geneticist, you analyze Talltwin’s DNA and find that he is homozygous for thalassemia. Assuming the disease is autosomal recessive, what can you deduce about the genotype of Mr. and Mrs. Talltwin?
a. Dad Talltwin is a carrier of the disease, and Mom Talltwin is normal.
b. Mom Talltwin is a carrier of the disease, and Dad Talltwin is normal.
c. Dad Talltwin is homozygous, and Mom Talltwin is normal.
d. Mom Talltwin is homozygous, and Dad Talltwin is normal.
e. Both Mom and Dad Talltwin are carriers of the disease.

A

e. Both Mom and Dad Talltwin are carriers of the disease.

How well did you know this?
1
Not at all
2
3
4
5
Perfectly
41
Q

A 6-year-old boy visits his physician because his parents have noticed autistic behavior and speech problems. The mother’s family does have a history of mental retardation. Therefore, the physician suggested a genetic screen of the fragile X mental retardation 1 (FMR1) gene for fragile X syndrome. Polymerase chain reaction (PCR) revealed borderline fragile X syndrome. What situation most likely explains this result?
A. A complete loss of fragile X mental retardation protein (FMRP)
B. An FMR1 gene CGG repeat expansion of 60 with partial DNA methylation
C. An FMR1 gene CGG repeat expansion of 230 with minor DNA methylation
D. An FMR1 gene CGG repeat expansion of 280 with complete DNA methylation

A

C. An FMR1 gene CGG repeat expansion of 230 with minor DNA methylation

How well did you know this?
1
Not at all
2
3
4
5
Perfectly
42
Q

Phil Hardy has decided to train for an upcoming marathon. Nearing the age of 50, Phil figures that after he trains he should be able to maintain a 9 minute-per-mile pace, which would mean that he would finish the race in approximately 4 hours. Given that he would be adequately hydrating himself at the various water stations along the way, as he is about to finish the 26 mile 385 yard course, what is the primary fuel that his leg muscles would be using?
a. Fatty acids from the blood
b. Glycerol from the blood
c. Glycogen stored in muscle
d. Glucose from the blood
e. Ketone bodies from the blood

A

a. Fatty acids from the blood

How well did you know this?
1
Not at all
2
3
4
5
Perfectly
43
Q

A postoperative patient on intravenous fluids develops lesions in the mouth (angular stomatitis). Urinalysis indicates an excretion of 15 μg riboflavin/mg creatinine, which is abnormally low. Which of the following TCA cycle enzymes is most likely to be affected?
a. Citrate synthase
b. Isocitrate dehydrogenase
c. Fumarase
d. Malate dehydrogenase
e. Succinate dehydrogenase

A

e. Succinate dehydrogenase

How well did you know this?
1
Not at all
2
3
4
5
Perfectly
44
Q

After excessive drinking over an extended period of time while eating poorly, a middle-aged man is admitted to the hospital with “high output” heart failure. Which of the following enzymes is most likely inhibited?
a. Aconitase
b. Citrate synthase
c. Isocitrate dehydrogenase
d. α-Ketoglutarate dehydrogenase
e. Succinate thiokinase

A

e. Succinate thiokinase

How well did you know this?
1
Not at all
2
3
4
5
Perfectly
45
Q

A full-term female infant failed to gain weight and showed metabolic acidosis in the neonatal period. A physical examination at 6 months showed failure to thrive, hypotonia, small muscle mass, severe head lag, and a persistent acidosis (pH 7.0 to 7.2). Blood lactate, pyruvate, and alanine were greatly elevated. Treatment with thiamine did not alleviate the lactic acidosis. Which of the following enzymes is most likely deficient in this patient?
a. Alanine aminotransferase
b. Phosphoenolpyruvate carboxykinase
c. Pyruvate carboxylase
d. Pyruvate dehydrogenase
e. Pyruvate kinase

A

d. Pyruvate dehydrogenase

How well did you know this?
1
Not at all
2
3
4
5
Perfectly
46
Q

A 3-month-old male infant developed seizures and progressively worsened, showing hypotonia, psychomotor retardation, and poor head control. He had lactic acidosis and an elevated plasma pyruvate level, both more than seven times the normal amount. Pyruvate carboxylase activity was measured using extracts of fibroblasts and was found to be less than 1 percent of the normal level. Oral administration of which of the following amino acids would you recommend as the best therapy for this patient?
a. Alanine
b. Glutamine
c. Leucine
d. Lysine
e. Serine

A

b. Glutamine

How well did you know this?
1
Not at all
2
3
4
5
Perfectly
47
Q

A deficiency in thiamine (vitamin B1) would most likely lead to which of the following clinical manifestations?

a. A decrease in carboxylase enzyme activity
b. A decrease in serum lactate concentrations
c. A decrease in red blood cell transketolase activity
d. An increase in urinary methylmalonate
e. An increase in prothrombin time

A

c. A decrease in red blood cell transketolase activity

How well did you know this?
1
Not at all
2
3
4
5
Perfectly
48
Q

A 16-month-old girl was found to have ingested approimately 30 mL of an acetonitrile-based cosmetic nail remover when she vomited 15 minutes postingestion. The poison control center was contacted, but no treatment was recommended because it was confused with an acetone-based nail polish remover. The child was put to bed at her normal time, which was 2 hours postingestion. Respiratory distress developed sometime after the child was put to bed, and she was found dead the next morning.

Inhibition of which of the following enzymes was the most likely cause of this child’s death?
a. Cytochrome c reductase
b. Cytochrome oxidase
c. Coenzyme Q reductase
d. NADH dehydrogenase
e. Succinate dehydrogenase

A

b. Cytochrome oxidase

How well did you know this?
1
Not at all
2
3
4
5
Perfectly
49
Q

Which of the following best describes the reason for the latency of acetonitrile toxicity and why prompt treatment would have prevented this child’s respiratory distress and death?
a. Acetonitrile crosses the mitochondrial membrane slowly.
b. Acetonitrile induces hemolysis by inhibiting glucose 6-phosphate dehydrogenase.
c. Acetonitrile is only poorly absorbed by the intestinal system.
d. Complex IV of the electron transport system binds acetonitrile weakly.
e. Cytochrome P450 enzymes oxidize acetonitrile and slowly release cyanide

A

e. Cytochrome P450 enzymes oxidize acetonitrile and slowly release cyanide

How well did you know this?
1
Not at all
2
3
4
5
Perfectly
50
Q

Inhibition of oxidative phosphorylation by cyanide ion leads to increases in which of the following?
a. Gluconeogenesis to provide more glucose for metabolism
b. Transport of ADP into the mitochondria
c. Utilization of fatty acids substrates to augment glucose utilization
d. Utilization of ketone bodies for energy generation
e. Lactic acid in the blood causing acidosis

A

e. Lactic acid in the blood causing acidosis

How well did you know this?
1
Not at all
2
3
4
5
Perfectly
51
Q

Which of the following procedures best describes the emergency intervention for cyanide poisoning?
a. Decrease the partial pressure of oxygen
b. Treatment with nitrites to convert hemoglobin to methemoglobin
c. Treatment with thiosulfate to form thiocyanate
d. Use of Mucomyst (N-acetylcysteine) taken orally

A

b. Treatment with nitrites to convert hemoglobin to methemoglobin

How well did you know this?
1
Not at all
2
3
4
5
Perfectly
52
Q

Respiratory distress is induced on rotenone exposure because it inhibits the complex that catalyzes which of the following?
a. Electron transfer from NADH to coenzyme Q
b. Oxidation of coenzyme Q
c. Reduction of cytochrome c
d. Electron transfer from cytochrome c to cytochrome a1 /a3
e. Electron transfer from cytochrome a1 /a3 to oxygen

A

a. Electron transfer from NADH to coenzyme Q

How well did you know this?
1
Not at all
2
3
4
5
Perfectly
53
Q

The major metabolic consequence of perturbation of the electron transfer in mitochondria is which of the following?
a. Increased production of NADPH
b. Increased oxidation of NADH
c. Increased reduction of O2 to H2O
d. Decreased regeneration of NAD+
e. Decreased reduction of FAD

A

d. Decreased regeneration of NAD+

How well did you know this?
1
Not at all
2
3
4
5
Perfectly
54
Q

Laboratory data show a remarkably increased carboxyhemoglobin level in the blood. The best explanation for this finding is that CO has which of the following effects?
a. It increases the hydrogen ion concentration causing oxyhemoglobin to precipitate
b. It changes the valence state of iron in hemoglobin
c. It competitively displaces O2 from oxyhemoglobin
d. It converts myoglobin to carboxyhemoglobin at a rapid rate
e. It prevents transfer of O2 across the alveolar membranes

A

c. It competitively displaces O2 from oxyhemoglobin

How well did you know this?
1
Not at all
2
3
4
5
Perfectly
55
Q

Which of the following treatment strategies is the most effective remediation for CO poisoning?
a. Removal from CO source
b. Removal from CO source and administration of 100 percent O2
c. Administration of 5 to 7 percent CO2 to stimulate respiration followed by 100 percent O2
d. Administration of 5 to 7 percent CO2 to stimulate respiration
e. Removal from CO source and administration of 5 to 7 percent CO2 to stimulate respiration

A

b. Removal from CO source and administration of 100 percent O2

How well did you know this?
1
Not at all
2
3
4
5
Perfectly
56
Q

In addition to forming carboxyhemoglobin, the toxic effects of CO include inhibition of which of the following enzymes involved in oxidation-reduction reactions?
a. NADPH dehydrogenase
b. Coenzyme Q reductase
c. Cytochrome c reductase
d. Succinate dehydrogenase
e. Cytochrome oxidase

A

e. Cytochrome oxidase

How well did you know this?
1
Not at all
2
3
4
5
Perfectly
57
Q

A young and otherwise healthy student undergoing preparation for a simple surgical procedure was noticed to have an elevating temperature and respiratory rate with muscle rigidity following the onset of anesthesia using halothane and succinylcholine. Laboratory findings revealed elevated levels of calcium, hydrogen ion, pyruvate, and lactate. The diagnosis was malignant hyperthermia.

The muscle rigidity observed in this patient is most likely prompted by which of the following?
a. The patient’s fear of surgery
b. Increased levels of hydrogen ion, causing the muscles to become immobile
c. Increased Ca2+ levels in muscle tissue, triggering muscle contraction
d. Increased pyruvate and lactate, causing precipitation of muscle protein
e. Increased body temperature

A

c. Increased Ca2+ levels in muscle tissue, triggering muscle contraction

How well did you know this?
1
Not at all
2
3
4
5
Perfectly
58
Q

Rapidly elevating body temperature was observed in this patient. The underlying cause for this pyretic episode is which of the following?
a. Hypothalamic upward adjustment of body temperature set point in response to cold operating room temperature
b. Muscles producing heat from exertion of contraction heat
c. Uncoupling proteins allowing dissipation of the mitochondrial hydrogen ion gradient releasing energy as heat
d. Metabolism of fatty acids from lipid storage depots releasing heat
e. Elevated consumption of ATP to support muscle contraction releasing heat

A

c. Uncoupling proteins allowing dissipation of the mitochondrial hydrogen ion gradient releasing energy as heat

How well did you know this?
1
Not at all
2
3
4
5
Perfectly
59
Q

Which of the following best describes the mechanism of action of dantrolene in the treatment of MH?
a. Decrease of Ca2+ release
b. Reduction of body temperature with hypothalamic set temperature
c. Effect on mitochondrial ATP production
d. Nuclear transcription attenuation
e. Recoupling of the sodium and ATP channels

A

a. Decrease of Ca2+ release

How well did you know this?
1
Not at all
2
3
4
5
Perfectly
60
Q

Prior to a race, many marathon runners will try to increase their glycogen concentrations by loading up with foods with a high starch content, such as pasta. α-Amylase secreted by the pancreas will digest the starch into which of the following major products?
a. Amylose, amylopectin, and maltose
b. Glucose, galactose, and fructose
c. Glucose, sucrose, and maltotriose
d. Limit dextrins, maltose, and maltotriose
e. Limit dextrins, lactose, and sucrose

A

d. Limit dextrins, maltose, and maltotriose

How well did you know this?
1
Not at all
2
3
4
5
Perfectly
61
Q

A 3-month-old infant presents with hepatosplenomegaly and failure to thrive. A liver biopsy reveals glycogen with an abnormal, amylopectinlike structure with long outer chains. Which of the following enzymes would most likely be deficient?
a. α-Amylase
b. Branching enzyme
c. Debranching enzyme
d. Glycogen phosphorylase
e. Glycogen synthase

A

b. Branching enzyme

How well did you know this?
1
Not at all
2
3
4
5
Perfectly
62
Q

A 3-year-old Caucasian female presents with chronic diarrhea and a failure to thrive. Stools were oily. History reveals that she was breastfed and had no problems until she was weaned. Which of the enzymes would be expected to be deficient following stimulation with secretin?
a. Cholesteryl esterase
b. Gastric lipase
c. Hormone sensitive lipase
d. Lipoprotein lipase
e. Pancreatic lipase

A

e. Pancreatic lipase

How well did you know this?
1
Not at all
2
3
4
5
Perfectly
63
Q

A female infant appeared normal at birth but developed signs of liver disease and muscular weakness at 3 months. She had periods of hypoglycemia, particularly on awakening. Examination revealed an enlarged liver. Laboratory analyses following fasting revealed ketoacidosis, blood pH 7.25, and elevations in both alanine transaminase (ALT) and aspartate transaminase (AST). Administration of glucagon following a carbohydrate meal elicited a normal rise in blood glucose, but glucose levels did not rise when glucagon was administered following an overnight fast. Liver biopsy revealed an increase in the glycogen content (6 percent of wet weight).

In which of the following enzymes is a genetic deficiency most likely for this patient?
a. Branching enzyme
b. Debranching enzyme
c. Glucose-6-phosphatase
d. Glycogen synthase
e. Muscle phosphorylase

A

b. Debranching enzyme

How well did you know this?
1
Not at all
2
3
4
5
Perfectly
64
Q

A female infant appeared normal at birth but developed signs of liver disease and muscular weakness at 3 months. She had periods of hypoglycemia, particularly on awakening. Examination revealed an enlarged liver. Laboratory analyses following fasting revealed ketoacidosis, blood pH 7.25, and elevations in both alanine transaminase (ALT) and aspartate transaminase (AST). Administration of glucagon following a carbohydrate meal elicited a normal rise in blood glucose, but glucose levels did not rise when glucagon was administered following an overnight fast. Liver biopsy revealed an increase in the glycogen content (6 percent of wet weight).

To prevent the frequent episodes of hypoglycemia, which of the following dietary supplements would be most appropriate for this patient?
a. Casein (milk protein)
b. Fish oil
c. Fructose
d. Lactose
e. Uncooked cornstarch

A

e. Uncooked cornstarch

How well did you know this?
1
Not at all
2
3
4
5
Perfectly
65
Q

A 17-year-old male presents complaining of an inability to perform strenuous exercise without bringing on painful muscle cramps and weakness. He indicated that mild to moderate exercise resulted in no problems. When he was administered an ischemic exercise test, his serum lactate concentrations did not increase significantly. A deficiency in which of the following enzymes is most likely the cause of the patient’s muscle cramps?
a. Carnitine palmitoyl transferase II
b. Glucose-6-phosphatase
c. Glycogen phosphorylase
d. Glycogen synthase
e. Very long chain acyl-CoA dehydrogenase

A

c. Glycogen phosphorylase

How well did you know this?
1
Not at all
2
3
4
5
Perfectly
66
Q

A 23-year-old male has been vigorously working on the yard and begins to feel slightly light-headed from hypoglycemia. He drinks a can of soda and is aware of the competition for the glucose to be stored in his liver as glycogen versus used as energy in his muscles. What is the best explanation regarding the fate of the glucose in the soda?
a. The lower Km of hexokinase versus the Km of glucokinase will tilt the glucose toward glycolysis
b. The bolus of glucose via the soda will lead to a higher glucose level, inducing storage of the glucose into glycogen in the liver.
c. The muscle is using high levels of glucose, leading to an increased level of glucose 6-phosphate thus inhibiting glucokinase.
d. The glucose will be equally used by muscle for metabolism and liver for glycogen storage.

A

a. The lower Km of hexokinase versus the Km of glucokinase will tilt the glucose toward glycolysis

How well did you know this?
1
Not at all
2
3
4
5
Perfectly
67
Q

During an extended period of exercise, the enzymes involved in the glycolytic pathway in muscle tissue are actively breaking down glucose to provide the muscle energy. The liver, to maintain blood glucose levels, is synthesizing glucose via the gluconeogenic pathway. Which of the following enzymes involved in these pathways would be most likely to exhibit Michaelis–Menten kinetics, that is, have a hyperbolic curve when plotting substrate concentration versus velocity of the reaction?

a. Fructose-1,6-bisphosphatase
b. Hexokinase
c. Lactate dehydrogenase
d. Phosphofructokinase 1
e. Pyruvate kinase

A

c. Lactate dehydrogenase

How well did you know this?
1
Not at all
2
3
4
5
Perfectly
68
Q

A known alcoholic is found lying semiconscious at the botton of a stairwell with a broken arm by his landlady, who called an ambulance to take him to the emergency room. Initial laboratory studies showed a relatively large anion gap of 34 (normal = 9 to 15). His blood alcohol was elevated at 245 mg/dL (intoxication level = 150 to 300 mg/dL), and his blood glucose was 38 mg/dL (low normal). The patient’s large anion gap and hypoglycemia can best be explained by which of the following?
A. Decreased secretion of glucagon
B. Increased secretion of insulin
C. Increased urination resulting from the diuretic effect of alcohol
D. Inhibition of dehydrogenase enzymes by NADH
E. Inhibition of glycogenolysis by ethanol

A

D. Inhibition of dehydrogenase enzymes by NADH

How well did you know this?
1
Not at all
2
3
4
5
Perfectly
69
Q

A young man with normocytic anemia, jaundice, and splenomegaly was diagnosed as having RBC pyruvate kinase deficiency after a peripheral blood smear showed spiculated cells.

Since in this patient pyruvate kinase is abnormal not only is less pyruvate made but intermediates above pyruvate in the glycolytic pathway build up slowing the pathway. Which of the following products may not be made in the appropriate amounts in the RBC because of the deficiency of pyruvate?
a. Glucose
b. Oxaloacetate
c. acetyl-CoA
d. Lactate

A

d. Lactate

How well did you know this?
1
Not at all
2
3
4
5
Perfectly
70
Q

A young man with normocytic anemia, jaundice, and splenomegaly was diagnosed as having RBC pyruvate kinase deficiency after a peripheral blood smear showed spiculated cells.

In the RBCs of the patient described above, which of the following would be expected?
a. ADP to ATP ratios would be elevated above normal.
b. NADP+ would increase relative to NADPH.
c. Ribulose 5-phosphate levels would decrease.
d. NADH to NAD+ ratios would decrease.
e. Methemoglobin levels would increase.

A

a. ADP to ATP ratios would be elevated above normal.

How well did you know this?
1
Not at all
2
3
4
5
Perfectly
71
Q

The glycolytic pathway is a multistep process by which glucose is broken down to a three-carbon metabolite. Some of the steps are listed below:
1. Conversion of 3-phosphoglycerate to 2-phosphoglycerate
2. Conversion of phosphoenolpyruvate to pyruvate
3. Conversion of glyceraldehyde 3-phosphate to 1,3- bisphosphoglycerate
4. Conversion of glucose to glucose 6-phosphate
5. Conversion of fructose 6-phosphate to fructose 1,6-bisphosphate

A
  1. Conversion of glucose to glucose 6-phosphate
  2. Conversion of fructose 6-phosphate to fructose 1,6-bisphosphate
  3. Conversion of glyceraldehyde 3-phosphate to 1,3-bisphosphoglycerate
  4. Conversion of 3-phosphoglycerate to 2-phosphoglycerate
  5. Conversion of phosphoenolpyruvate to pyruvate
How well did you know this?
1
Not at all
2
3
4
5
Perfectly
72
Q

A 22-year-old soldier collapses from dehydration during maneuvers in the desert and is sent to a military hospital. Prior to enlisting, a physician observed a high level of glucose in his urine during an examination. At first, he was not allowed to enlist because he was suspected of being a diabetic. Further tests, however, determined that his insulin level was normal. A glucose tolerance test exhibited a normal pattern. Laboratory tests following his dehydration episode repeat the previous findings, but further testing of the urine reveals that only D-glucose is elevated. Other sugars were not elevated. This patient’s elevated urinary glucose and his dehydration episode are caused by a deficiency in which of the following?
a. GLUT 2
b. GLUT 4
c. Insulin receptor
d. SGLT1
e. SGLT2

A

e. SGLT2
Kidney glucose transporter

How well did you know this?
1
Not at all
2
3
4
5
Perfectly
73
Q

Your patient is a 7-month-old baby girl, the second child born to unrelated parents. She did not respond well to breast-feeding and was changed entirely to a formula based on cow’s milk at 4 weeks. Between 7 and 12 weeks of age, she was admitted to the hospital twice with a history of screaming after feeding, but was discharged after observation without a specific diagnosis. Elimination of cow’s milk from her diet did not relieve her symptoms; her mother reported that the screaming bouts were worse after the child drank juice and that she frequently had gas and a distended abdomen. Analysis of a liver needle biopsy did not reveal any liver enzyme deficiencies. Overall, the girl is thriving (weight greater than 97th percentile) with no abnormal findings on physical examination.

If a biopsy of intestinal tissue were obtained from your patient and analyzed, which of the following would most likely be deficient or defective?
a. GLUT 2
b. GLUT 5
c. Isomaltase
d. Lactase
e. SGLT1

A

b. GLUT 5

How well did you know this?
1
Not at all
2
3
4
5
Perfectly
74
Q

A 24-year-old African-American female presents with complaints of intestinal bloating, gas, cramps, and diarrhea following a meal including dairy products. A lactose-tolerance test confirms your suspicion that she had a deficiency of lactase in her intestine. Which of the following dairy products could you recommend that would be least likely to cause her difficulties in the future?
a. Condensed milk
b. Cottage cheese
c. Ice cream
d. Skim milk
e. Yogurt

A

e. Yogurt

How well did you know this?
1
Not at all
2
3
4
5
Perfectly
75
Q

A patient with type I diabetes mellitus has fasting and postprandial blood glucose levels that are frequently above the normal range despite good compliance with his insulin therapy. He was referred to a dietician that specialized in diabetic patients. The patient was recommended to incorporate foods high in dietary fiber. Which of the following dietary fibers would be most helpful in maintaining a normal blood glucose level?
a. Cellulose
b. Hemicellulose
c. Lignins
d. Pectins

A

d. Pectins

How well did you know this?
1
Not at all
2
3
4
5
Perfectly
76
Q

Some individuals complain of flatulence following a meal plentiful in beans, peas, soybeans, or other leguminous plants. All legumes contain the oligosaccharides raffinose and stachyose that contain glycosidic linkages that are poorly hydrolyzed by intestinal enzymes but are good sources of energy for intestinal bacteria that convert these sugars to H2. Which of the following glycosidic bonds are contained in raffinose and stachyose and are not hydrolyzed by our intestinal enzymes but can be by intestinal flora?
a. Galactose (α1→6) glucose
b. Galactose (β1→4) glucose
c. Glucose (β1→2) fructose
d. Glucose (α1→4) glucose
e. Glucose (β1→4) glucose

A

a. Galactose (α1→6) glucose

How well did you know this?
1
Not at all
2
3
4
5
Perfectly
77
Q

Cellulose is the most abundant polysaccharide and is an important structural component of cell walls. Strict vegetarians consume a large amount of cellulose, but it is not a source of energy because it is indigestible by the human intestinal tract.

Cellulose is indigestible because it contains which of the following glycosidic bonds?
a. Galactose (β1→4) glucose
b. Galactose (β1→6) galactose
c. Glucose (α1→4) glucose
d. Glucose (β1→2) fructose
e. Glucose (β1→4) glucose

A

e. Glucose (β1→4) glucose

How well did you know this?
1
Not at all
2
3
4
5
Perfectly
78
Q

A 27-year-old man has been rushed to the emergency room following his sudden collapse and entry into a state of unconsciousness. Examination of personal belongings revealed the patient is an insulin-dependent diabetic.

A rapid decline in which of the following humoral factors likely triggered the sudden collapse of the patient?
a. Insulin
b. Glucagon
c. Fatty acids
d. Glucose
e. Triglyceride

A

d. Glucose

How well did you know this?
1
Not at all
2
3
4
5
Perfectly
79
Q

Which of the following is least likely to contribute to the hyperglycemia associated with uncontrolled type I diabetes?
a. Decreased skeletal muscle glucose uptake
b. Decreased adipose lipogenesis
c. Increased adipose lipolysis
d. Increased hepatic gluconeogenesis
e. Increased skeletal muscle glycogenolysis

A

e. Increased skeletal muscle glycogenolysis

How well did you know this?
1
Not at all
2
3
4
5
Perfectly
80
Q

Which of the following changes in hepatic metabolism best explains the increased incidence of ketoacidosis observed in type I diabetes?
a. Increased glucose uptake
b. Increased protein synthesis
c. Increased lipoprotein synthesis
d. Increased B-oxidation
e. Increased glycogen breakdown

A

d. Increased B-oxidation

How well did you know this?
1
Not at all
2
3
4
5
Perfectly
81
Q

A middle-aged man, after an episode of retrosternal chest pressure with radiation to the neck and associated nausea and diaphoresis while at rest, was given a diagnosis of unstable angina and possibly myocardial infarction. Subsequent laboratory findings of increased serum levels of troponin I and cardiac enzymes were consistent with the diagnosis.

If the patient described above did indeed have angina, which of the following change(s) in metabolism in the affected area would occur?
a. Increased oxidative phosphorylation
b. Increased rate of fatty acid oxidation
c. Increased conversion of pyruvate to acetyl-CoA
d. Increased formation of lactate
e. Increased use of ketone bodies

A

d. Increased formation of lactate

How well did you know this?
1
Not at all
2
3
4
5
Perfectly
82
Q

A middle-aged man, after an episode of retrosternal chest pressure with radiation to the neck and associated nausea and diaphoresis while at rest, was given a diagnosis of unstable angina and possibly myocardial infarction. Subsequent laboratory findings of increased serum levels of troponin I and cardiac enzymes were consistent with the diagnosis.

In the case described above an ischemic episode very likely occurred. What changes in glucose metabolism would be observed?
a. The overall rate of glucose utilization would decrease.
b. Pyruvate kinase is allosterically inhibited.
c. The rate of ATP production in the cytosol increases.
d. NADH is reoxidized to NAD+via the glycerol 3- phosphate shuttle.

A

c. The rate of ATP production in the cytosol increases.

How well did you know this?
1
Not at all
2
3
4
5
Perfectly
83
Q

If the patient described does indeed have angina, his is a focal lesion wherein only one portion of heart muscle is affected while other portions still receive adequate oxygen enabling the malate-aspartate shuttle to function. Which of the following is involved in this shuttle to transfer electrons across the mitochondrial barrier?
a. Malate-aspartate antiporter
b. Malate-a-ketoglutarate antiporter
c. Glutamate-a-ketoglutarate antiporter
d. Aspartate-a-ketoglutarate symporter
e. Malate-glutamate symporter

A

b. Malate-a-ketoglutarate antiporter

How well did you know this?
1
Not at all
2
3
4
5
Perfectly
84
Q

A couple is seen in your office for genetic counseling regarding Tay-Sachs disease. They are very knowledgeable and request more information about the specific enzyme that is defective in this disease. You explain that Tay-Sachs results from the lack of an enzyme activity necessary for which of the following?
a. Removal of N-acetylgalactosamine from ganglioside GM2
b. Addition of N-acetylgalactosamine to ganglioside GM2
c. Removal of the disaccharide galactose-N-acetylgalactosamine from ganglioside GM2
d. Addition of the disaccharide galactose-N-acetylgalactosamine to ganglioside GM2
e. Removal of a galactose residue from ganglioside GM2

A

a. Removal of N-acetylgalactosamine from ganglioside GM2

How well did you know this?
1
Not at all
2
3
4
5
Perfectly
85
Q

Tay-Sachs disease involves the metabolism of gangliosides. Gangliosides are composed of a ceramide backbone with at least which one of the following?
a. Phosphorylated sugar residue
b. Glucose residue
c. Galactose residue
d. Sialic acid residue
e. Fructose residue

A

d. Sialic acid residue

How well did you know this?
1
Not at all
2
3
4
5
Perfectly
86
Q

The genetic disease which results from a mutation in the gene coding for the enzyme hexosaminidase (b-N-acetylhexosaminidase) is called what?
a. Huntington disease
b. Lesch-Nyhan syndrome
c. Tay-Sachs disease
d. Amyotrophic lateral sclerosis
e. Neurofibromatosis

A

c. Tay-Sachs disease

How well did you know this?
1
Not at all
2
3
4
5
Perfectly
87
Q

A 5-year old boy is seen by a pediatrician because his parents are concerned about his aggressive behavior, hyperactivity, and a loss of language skills. He also has recently become increasingly unsteady on his feet and has experienced a recent seizure. Slight facial feature coarsening is noted. In which of the following processes is this child most likely to have a disorder?
a. Mobilization of glycogen
b. Gluconeogenesis
c. Salvage of purine bases
d. Degradation of glycosaminoglycans
e. Cholesterol metabolism

A

d. Degradation of glycosaminoglycans
The patient is exhibiting the classic symptoms of Sanfilippo syndrome, which is a deficiency in one of four different lysosomal enzymes that breakdown glycosaminoglycans leading to the buildup of heparan sulfate and dermatan sulfate in lysosomes.

How well did you know this?
1
Not at all
2
3
4
5
Perfectly
88
Q

A15-month old white female is brought to the pediatrician because of recurrent upper respiratory tract infection. During physical exam the girl is noted to have a short stature, some clouding of the corneas, coarse facial features, and an enlarged tongue. She also appears to have some hearing loss and other developmental delays. The pediatrician suspects the child has a mucopolysaccharidosis. Which of the following is she least likely to have?
a. Hurler syndrome (MPS I)
b. Hunter syndrome (MPS II)
c. Morquio syndrome (MPS IV)
d. Sly syndrome (MPS VII)
e. Sanfilippo syndrome (MPS III)

A

b. Hunter syndrome (MPS II)

How well did you know this?
1
Not at all
2
3
4
5
Perfectly
89
Q

A 3-year-old male with coarse facial features, progressive loss of motor skills, hepatosplenomegaly and chronic diarrhea is suspected of having Hunter syndrome (MPS II). Which of the following monosaccharide residues would be expected to be found at the nonreducing end of glycosaminoglycans in this patient’s urine?
a. N-Acetylglucosamine
b. N-Acetylgalactosamine
c. Glucuronate
d. Iduronate
e. Iduronate 2-sulfate

A

e. Iduronate 2-sulfate

How well did you know this?
1
Not at all
2
3
4
5
Perfectly
90
Q

A patient presents in your office with very high levels of serum cholesterol. After a series of tests, you conclude that the patient has high circulating levels of LDL cholesterol, but has normal levels of the liver LDL receptor. One possible explanation for this observation is which of the following?
a. The patient has a mutated form of apoprotein B-100
b. The inability to selectively remove cholesterol from the LDL complex
c. The absence of the enzyme lipoprotein lipase
d. Decreased levels of acyl-CoA: cholesterol acyltransferase
e. Altered phosphorylation of the LDL receptor

A

a. The patient has a mutated form of apoprotein B-100

How well did you know this?
1
Not at all
2
3
4
5
Perfectly
91
Q

A patient with hereditary type I hyperlipidemia presents with elevated levels of chylomicrons and VLDL triglycerides in the blood. The main function of the chylomicrons in circulation is to do which of the following?
a. Transport lipids from the liver
b. Transport dietary lipids from the intestine to target tissues
c. Transport cholesterol from IDL to LDL
d. Act as a receptor for triacylglycerols in the liver
e. Bind cholesterol esters exclusively

A

b. Transport dietary lipids from the intestine to target tissues

How well did you know this?
1
Not at all
2
3
4
5
Perfectly
92
Q

Free cholesterol can affect cholesterol metabolism in the body by inhibiting cholesterol biosynthesis. The step at which free cholesterol inhibits its biosynthesis is by inhibiting which of the following processes?
a. Cyclizing of squalene to form lanosterol
b. Reduction of 7-dehydrocholesterol to form cholesterol
c. Formation of mevalonate from hydroxymethylglutaryl-CoA
d. Kinase that phosphorylates hydroxymethylglutaryl-CoA reductase
e. Condensation of acetyl-CoA and acetoacetyl-CoA to form hydroxymethylglutaryl-CoA

A

c. Formation of mevalonate from hydroxymethylglutaryl-CoA

How well did you know this?
1
Not at all
2
3
4
5
Perfectly
93
Q

A patient presents in your office with very high levels of serum cholesterol. He states that he has tried to follow the diet and exercise regimen you gave him last year. You decide that this patient would benefit from a drug such as Lipitor (atorvastatin). This class of drugs is effective in treating hypercholesterolemia because it has what effect?
a. Stimulates phosphorylation of the b-hydroxy-b-methylglutaryl-CoA reductase enzyme
b. Decreases the stability of the b-hydroxy-b-methylglutaryl-CoA reductase protein
c. Binds cholesterol preventing it from being absorbed by the intestine
d. Directly prevents the deposition of cholesterol on artery walls
e. Inhibits the enzyme b-hydroxy-b-methylglutaryl-CoA reductase

A

e. Inhibits the enzyme b-hydroxy-b-methylglutaryl-CoA reductase

How well did you know this?
1
Not at all
2
3
4
5
Perfectly
94
Q

The modification to bile salts that increases the working pH range and amphipathic nature of bile salts is:
a. 7a-Hydroxylation
b. Dehydroxylation by intestinal bacteria
c. Esterification
d. Conjugation to taurine or glycine

A

d. Conjugation to taurine or glycine

How well did you know this?
1
Not at all
2
3
4
5
Perfectly
95
Q

A new drug called CT2033, a corticosteroid, has reached the clinical trial stage. This drug was designed to treat inflammation but seems to also cause an undesired side effect where there is disturbance of cholesterol and bile acid homeostasis. Which of the following is least likely to explain the side effects caused by CT2033?
a. It downregulates the expression of a hepatobiliary bile acid transporter gene.
b. It inhibits a hepatobiliary transporter protein decreasing bile acid secretion.
c. It competes with cholesterol for CYP7A1 binding.
d. It binds and inhibits pancreatic lipases.

A

d. It binds and inhibits pancreatic lipases.

How well did you know this?
1
Not at all
2
3
4
5
Perfectly
96
Q

A 53-year-old male patient with elevated levels of low-density lipoprotein (LDL) cholesterol, signs of premature cholesterol gallstone disease and substantially elevated triglycerides visited his physician for a follow-up to check his current status. The patient had received various statin, HMG-CoA-reductase inhibitors therapies for the past 2 years. However, after blood work done at this follow-up visit, complications had still not subsided. This patient has similar problems as two of his siblings. Which of the following best explains this patients dyslipidemia?
a. An influx of abnormal phospholipids in the gallbladder as a result of ileal disease.
b. A loss of HMG-CoA reductase function
c. A loss of CYP7A1 (cholesterol 7α-hydroxylase) function
d. Elevated levels of ACAT

A

c. A loss of CYP7A1 (cholesterol 7α-hydroxylase) function

How well did you know this?
1
Not at all
2
3
4
5
Perfectly
97
Q

The coxibs, including celecoxib (Celebrex), are a recently developed class of nonsteroidal antiinflammatory drugs (NSAIDs). The coxibs show anti-inflammatory actions without affecting platelet function. These effects of the coxibs are best attributed to selective inhibition of which of the following?
a. The cytosolic isozyme of phospholipase A2 (cPLA2)
b. The cyclooxygenase activity of the “basal” prostaglandin H synthase isozyme (PGHS-1)
c. The cyclooxygenase activity of the “inducible” prostaglandin H synthase isoform (PGHS-2)
d. The microsomal isozyme of prostaglandin E synthase (mPGES-1)
e. Prostacyclin (PGI2) synthase (PGIS)

A

c. The cyclooxygenase activity of the “inducible” prostaglandin H synthase isoform (PGHS-2)

How well did you know this?
1
Not at all
2
3
4
5
Perfectly
98
Q

Prostaglandins comprise a family of oxygenated lipid signaling molecules derived from polyunsaturated fatty acids such as arachidonic acid. They are involved in regulating a number of cellular processes. Some of the prostaglandins act to increase vasodilation and levels of cAMP in cells, whereas others increase vaso- and bronchoconstriction and smooth muscle contraction. In the conversion of arachidonic acid to prostaglandins, the oxygenation step is accomplished by the enzyme that synthesizes which of the following compounds?
a. Prostaglandin D2
b. Prostaglandin E2
c. Prostaglandin F2a
d. Prostaglandin H2
e. Prostaglandin I2

A

d. Prostaglandin H2

How well did you know this?
1
Not at all
2
3
4
5
Perfectly
99
Q

Signaling via prostanoids begins by interaction of the prostanoid with its receptor. The receptor involved is usually located in which part of the cell?
a. Plasma membrane of a cell near the cell making the prostanoid
b. Nucleus of a cell in a different organ from the cell making the prostanoid
c. Endoplasmic reticulum of the cell making the prostanoid
d. Lysosomes of a cell circulating in the blood
e. Golgi of a cell circulating in the blood

A

a. Plasma membrane of a cell near the cell making the prostanoid

How well did you know this?
1
Not at all
2
3
4
5
Perfectly
100
Q

An 18-year-old male with sickle cell anemia develops severe right upper-abdominal pain radiating to his lower right chest and his right flank 36 hours prior to admission to the ER. Twelve hours following the onset of pain, he began to vomit intractably. In the past year he has had several episodes of mild back and lower extremity pain that he attributed to mild sickle cell crises. He reported that the present pain was not like his usual crisis pain. He also reports that his urine is the color of iced tea and his stool now has a light clay color. On examination, his temperature is slightly elevated, and heart rate is rapid. He is exquisitely tender to pressure over his right upper abdomen. The sclerae of his eyes are slightly yellowish in color. What is the most likely cause of this patient’s symptoms?
a. A cholesterol-rich gallstone
b. A defect in the synthesis of bile acids
c. A defect in heme synthesis
d. A gallstone rich in calcium bilirubinate
e. A sickle cell crisis brought on by overexertion

A

d. A gallstone rich in calcium bilirubinate

How well did you know this?
1
Not at all
2
3
4
5
Perfectly
101
Q

A patient has been on combination statin and cholestyramine therapy to lower his serum cholesterol levels. Prior to any surgery, this patient would be well advised to be supplemented with which of the following?
a. Vitamin A
b. Vitamin B12
c. Vitamin C
d. Vitamin K
e. Linolenic acid

A

d. Vitamin K

How well did you know this?
1
Not at all
2
3
4
5
Perfectly
102
Q

A 17-year-old female, whose parents were first cousins, presents to a neurologist because of recurring seizures despite being on anticonvulsive therapy. Nodules that appeared to be fatty deposits were present on her Achilles tendon and several of her joints. Plasma cholesterol concentrations were elevated, and an assay of plasma sterols indicated elevated cholestanol. Cultured skin fibroblasts did not contain any sterol 27- hydroxylase activity. A diagnosis of cerebrotendinous xanthomatosis, a genetic disease inherited in an autosomal fashion, was made. A deficiency in sterol 27-hydroxylase would lead to a decrease in the synthesis of which of the following compounds?
a. Chenodeoxycholate
b. Cortisol
c. 1,25 Dihydroxycholecalciferol
d. Estradiol
e. Testosterone

A

a. Chenodeoxycholate

How well did you know this?
1
Not at all
2
3
4
5
Perfectly
103
Q

Which of the following compounds directly inhibits the expression of the HMG-CoA reductase gene?
a. Squalene
b. HMG-CoA
c. Lanosterol
d. Isopentenyl pyrophosphate
e. Cholesterol

A

e. Cholesterol

How well did you know this?
1
Not at all
2
3
4
5
Perfectly
104
Q

You decide to treat a patient who has very high levels of serum cholesterol with the statin drug Lipitor (atorvastatin). You know that this drug acts in the metabolic pathway leading to the synthesis of cholesterol. The substrate for the enzyme inhibited by the statin drugs is which of the following?
a. Acetoacetyl-CoA
b. HMG-CoA
c. Farnesol pyrophosphate
d. Isopentenyl pyrophosphate
e. Mevalonate

A

b. HMG-CoA

How well did you know this?
1
Not at all
2
3
4
5
Perfectly
105
Q

Which of the following vitamins can be used in high doses to treat hypercholesterolemia?
a. Niacin
b. Riboflavin
c. Pyridoxine
d. Folic acid
e. Thiamine

A

a. Niacin

How well did you know this?
1
Not at all
2
3
4
5
Perfectly
106
Q

A teenage boy presents with moderate to severe epigastric pain. Physical examination reveals extensive eruptive xanthomas and hepatosplenomegaly. A blood sample reveals milky plasma. Which of the following is the most likely lipoprotein to be elevated in this patient’s plasma?
a. Chylomicrons
b. Chylomicron remnants
c. HDL
d. IDL
e. LDL

A

a. Chylomicrons

How well did you know this?
1
Not at all
2
3
4
5
Perfectly
107
Q

Laboratory results for a patient with uncontrolled Type I diabetes mellitus reveal hyperglycemia (634 mg/dL) and hypertriglyceridemia (498 mg/dL). The most likely cause of the hypertriglyceridemia in this patient is which of the following?
a. Deficiency in apoprotein C-II
b. Increased hepatic triglyceride synthesis
c. Decreased lipoprotein lipase activity
d. Deficiency in LDL receptors
e. Absence of hormone-sensitive lipase

A

c. Decreased lipoprotein lipase activity

108
Q

A25-year-old female was referred to a lipid research center for investigation of moderate hypertriglyceridemia because the plasma lipid and lipoprotein profiles showed abnormalities. Both HDL and LDL were more buoyant and showed elevations in TG content with the mass of TG approximately the same as that of cholesterol. A deficiency in which of the following is the most likely cause of this patient’s lipid abnormality?
a. Lecithin-cholesterol acyltransferase (LCAT)
b. Lipoprotein lipase
c. ApoproteinC-II
d. Hepatic lipase
e. Apoprotein B-100

A

d. Hepatic lipase

109
Q

An 8-month-old child presents with exhaustion, irritability, and malnutrition. The family history reveals poverty and inadequate nutrition in all members. The 8-month-old was fed diluted formula, and the tentative diagnosis of marasmus was made.

In cases of starvation many metabolic changes take place to meet the body’s metabolic demands. Which of the following illustrates starvation-triggered changes in intermediary metabolism?
a. Increased dependence of liver on glucose for its energy supply
b. Increased synthesis of proteins in muscle tissue
c. Increased use of ketone bodies for energy source in brain
d. Decreased mobilization of triglycerides by adipose tissue
e. Adaptation of red blood cells to use ketone bodies for energy

A

c. Increased use of ketone bodies for energy source in brain

110
Q
  1. Acetoacetyl-CoA + acetyl-CoA → B-hydroxy-B- methylglutaryl- CoA + CoA
  2. Acetoacetate + NADH → B-hydroxybutyrate + NAD+
  3. B-hydroxy-B-methylglutaryl-CoA + H2O → acetoacetate + acetyl-CoA
  4. Acetyl-CoA + acetyl-CoA → acetoacetyl-CoA + CoA
  5. Acetoacetate + succinyl-CoA → acetoacetyl-CoA + succinate

Using the above reactions, which of the following correctly describes the pathway of ketone body formation?
a. 3→2→1→4
b. 4→1→3→2
c. 4→2→3→1
d. 5→1→2→3
e. 5→2→3→1

A

b. 4→1→3→2

111
Q

During starvation muscle activity decreases, and muscle protein is broken down to provide a carbon source for the liver production of glucose via gluconeogenesis. Which of the following amino acids remains in the muscle cell to provide a source of energy for the muscle?
a. Alanine
b. Aspartate
c. Leucine
d. Glutamate
e. Threonine

A

c. Leucine

112
Q

An 8 1/2-month-old infant was admitted to the hospital in a coma and a temperature of 39.4°C (102.9°F). His pulse was elevated, his liver was enlarged, and an EEG was grossly abnormal. Since the infant could not retain milk given by gavage feeding, intravenous glucose was administered. He improved rapidly and came out of the coma in 24 hours. Analysis of his urine showed abnormally high amounts of glutamine and uracil, which suggested a high blood ammonium ion concentration. The laboratory confirmed this. Considering the data, which enzyme may be defective in this patient?
a. Arginase
b. Carbamoyl phosphate synthetase I
c. Glutamate dehydrogenase
d. Glutaminase
e. Ornithine transcarbamoylase

A

e. Ornithine transcarbamoylase

113
Q

A newborn male infant was diagnosed as having phenylketonuria (PKU), and immediately placed on diet low in phenylalanine (Phe); careful compliance with the diet and frequent monitoring of the patient’s plasma Phe level resulted in the level being maintained at the lower limit of the normal range. The patient appeared to be developing normally until 4 months of age, when he developed truncal hypotonia and spasticity of the limbs. Despite being on a low-phenylalanine diet, at 5 months the patient had several grand mal (epileptic) seizures. After an abnormal Phe-loading test, the patient’s urine was found to have a markedly elevated urinary biopterin concentration. Which of the following enzymes is most likely deficient in this patient?
a. Dihydropteridine reductase
b. GTP cyclohydrolase I
c. Phenylalanine hydroxylase
d. Tryptophan hydroxylase
e. Tyrosine hydroxylase

A

a. Dihydropteridine reductase

114
Q

A newborn male infant was diagnosed as having phenylketonuria (PKU), and immediately placed on diet low in phenylalanine (Phe); careful compliance with the diet and frequent monitoring of the patient’s plasma Phe level resulted in the level being maintained at the lower limit of the normal range. The patient appeared to be developing normally until 4 months of age, when he developed truncal hypotonia and spasticity of the limbs. Despite being on a low-phenylalanine diet, at 5 months the patient had several grand mal (epileptic) seizures. After an abnormal Phe-loading test, the patient’s urine was found to have a markedly elevated urinary biopterin concentration.

Which treatment regimen would be most beneficial to this patient in Question 112?
a. A low-Phe diet with biopterin supplementation
b. A low-Phe diet with cobalamin (vitamin B12) supplementation
c. A low-Phe diet + L-dopa (3,4-dihydroxyphenylalanine)
d. A low-Phe diet + L-dopa and 5-hydroxytryptophan
e. A diet completely free of Phe

A

d. A low-Phe diet + L-dopa and 5-hydroxytryptophan

115
Q

A 3-month-old boy presents with elevated levels of phenylalanine, para-hydroxyphenylpyruvate and phenylpyruvate in the serum. His skin color is pale. On your differential diagnosis is phenylketonuria. Which of the following would be consistent in such a case?
a. Elevated levels of homogentisic acid in the serum
b. A deficiency in vitamin B12 (cobalamin)
c. Elevated levels of pyridoxal phosphate in the serum
d. The urine in the boy’s diaper smells like fresh maple syrup
e. Phenylalanine hydroxylase activity is only 2 percent of normal

A

e. Phenylalanine hydroxylase activity is only 2 percent of normal

116
Q

A 1-year-old girl presents at your clinic the day after you saw the 3- month-old boy. The symptoms are the same so you order a test on phenylalanine hydroxylase to confirm your diagnosis of phenylketonuria. To your surprise the phenylalanine hydroxylase activity is well within the normal range. Which of the following might you check next to support your diagnosis?
a. Tyrosine: a-ketoglutarate transaminase
b. Tyrosinase
c. Homogentisic acid oxidase
d. Dihydropteridine reductase
e. Dopamine hydroxylase

A

a. Tyrosine: a-ketoglutarate transaminase

117
Q

Skin color is the aggregate result of the expression of a number of genes modified by ethnic origin and genetic inheritance. Hypopigmentation may be caused by which of the following?
a. Excess formation of melanin
b. Excess phenylalanine in the serum and tissues
c. Hyposecretion of melatonin
d. Excessive stimulation of tyrosinase
e. Low levels of para-hydroxyphenylpyruvate

A

b. Excess phenylalanine in the serum and tissues

118
Q

A 12-year-old girl is presented at the clinic with reports of frequent fainting and lethargy. The girl is 5 ft tall and weighs 80 lb. Skin fold tests show an abnormally low percentage of body fat mass.

Which of the following is least likely to be consistent with the patient’s symptoms?
a. Anorexia nervosa
b. Bulimia
c. Type I diabetes
d. Type II diabetes

A

d. Type II diabetes

119
Q

A 12-year-old girl is presented at the clinic with reports of frequent fainting and lethargy. The girl is 5 ft tall and weighs 80 lb. Skin fold tests show an abnormally low percentage of body fat mass.

Which of the following hormones is likely to be severely diminished in the patient described above?
a. Cortisol
b. Epinephrine
c. Glucagon
d. Insulin

A

d. Insulin

120
Q

A 12-year-old girl is presented at the clinic with reports of frequent fainting and lethargy. The girl is 5 ft tall and weighs 80 lb. Skin fold tests show an abnormally low percentage of body fat mass.

Which of the following metabolic fluxes would be most consistent in this patient?
a. Protein→amino acids
b. Glucose→fatty acids
c. Glucose→glycogen
d. Fatty acids→triacylglycerol

A

a. Protein→amino acids

121
Q

A patient admitted to the emergency room with nausea and vomiting showed low serum potassium, elevated blood enzymes, and acetaminophen blood level above 200 μg/mL. The patient was diagnosed with acetaminophen overdose. Acetaminophen is a widely used analgesic. What is the most probable explanation of how a high dose of acetaminophen might have led to a toxic condition?
a. Acetaminophen itself is toxic.
b. Acetaminophen is metabolized to a potential toxic product.
c. Acetaminophen is metabolized to a potential toxic product that is fully conjugated.
d. Acetaminophen is metabolized to a potential toxic product that is partially conjugated.
e. None of the above

A

d. Acetaminophen is metabolized to a potential toxic product that is partially conjugated.

122
Q

Glutathione is a critical tripeptide involved in conjugation reactions and in reactions that protect cells from reactive oxygen species. Which of the following components compose glutathione?
a. Glutamic acid, alanine, methionine
b. Glutamine, alanine, cysteine
c. Glutamate, glycine, cysteine
d. Alanine, glycine, cysteine
e. Methionine, glycine, cysteine

A

c. Glutamate, glycine, cysteine

123
Q

All amino acids are needed for the production of proteins in cells and for the synthesis of important biomolecules. Which of the following amino acids, all of which can be synthesized by the human body, must be taken in the diet because it is not synthesized in sufficient quantities to meet the body’s needs?
a. Asparagine
b. Glutamine
c. Methionine
d. Proline
e. Tyrosine

A

c. Methionine

124
Q

A young child is in an automobile accident that requires surgical intevention and substantial recovery time in the hospital. A consultation with a nutritionist results in a specific dietary plan. The plan included the supplementation of an amino acid that is typically considered a nonessential amino acid. Which of the following amino acids is an essential amino acid under conditions of enhanced growth or surgical recovery?
a. Alanine
b. Arginine
c. Glycine
d. Serine
e. Tyrosine

A

b. Arginine

125
Q

As part of a standard neonatal screen, an infant is diagnosed with a loss of function genetic defect in the enzyme phenylalanine hydroxylase. Defects in this enzyme can result in a condition known as phenylketonuria (PKU), which results from the toxic effects of phenylalanine derived phenylketones. Fortunately, this condition can be managed by regulating the amount of phenylalanine provided in the diet. Which of the following nonessential amino acids will need to be supplied in the diet of this infant?
a. Alanine
b. Aspartate
c. Glycine
d. Serine
e. Tyrosine

A

e. Tyrosine

126
Q

A patient with a tentative diagnosis of cobalamin deficiency is awaiting a comprehensive blood analysis. Which of the following perturbations would NOT fit with the putative diagnosis?
a. Elevated levels of methylmalonic acid
b. Elevated levels of propionic acid
c. Elevated levels of para-hydroxyphenylpyruvate
d. Decreased levels of erythrocytes
e. Elevated levels of megaloblasts

A

c. Elevated levels of para-hydroxyphenylpyruvate

127
Q

In the role of cobalamin in metabolism. Which of the following statements is true?
a. Cyanocobalamin is the principal form of cobalamin used physiologically.
b. Cobalamin is equally active with an iron cofactor.
c. Enterocytes produce an intrinsic factor required for uptake of cobalamin in the gut.
d. Cobalamin is transported in the blood to tissues by proteins named transcobalamins.
e. Cobalamin is active in its 3+ oxidation state.

A

d. Cobalamin is transported in the blood to tissues by proteins named transcobalamins.

128
Q

Megaloblastic anemia has two most likely causes, deficiency of folate and deficiency of cobalamin. Often treatment of patients with cobalamin deficiency improves in terms of their hematologic features with treatment with folate but not in their neurologic symptoms. What is the most likely explanation for this explanation?
a. Cobalamin deficiencies are not serious
b. Excess folate blunts the trapping of folate as N5-methyltetrahydrofolate
c. Folate in high concentrations can serve as cofactor for the conversion of homocysteine to methionine
d. Excess folate directly inhibits the destruction of red blood cells
e. Excess folate stimulates erythropoietic tissues to synthesize cobalamin in situ

A

b. Excess folate blunts the trapping of folate as N5-methyltetrahydrofolate

129
Q

A patient presents with extreme swelling and tenderness in the joints of the extremities. Examination of synovial fluid extracted from the big toe reveals the presence of urate crystals and confirms the diagnosis of gouty arthritis. The drug allopurinol is prescribed to inhibit which of the following enzymes?
a. Adenosine deaminase
b. AMP deaminase
c. Nucleoside phosphorylase
d. Uricase
e. Xanthine oxidase

A

e. Xanthine oxidase

130
Q

Hyperuricemia (gout) is a clinical condition characterized by elevated levels of uric acid that lead to the formation of sodium urate crystals that are found primarily in the joints of the extremities. Which of the following factors contributes most to the formation of urate crystals in the extremities?
a. Decreased blood flow
b. Decreased temperature
c. Exposure to sunlight
d. Increased blood flow
e. Increased mobility

A

b. Decreased temperature

131
Q

Inherited defects in components of purine catabolism and salvage are associated with various conditions and syndromes. The enzyme hypoxanthine-guanine phosphoribosyltransferase (HGPRT) is a key enzyme in the purine salvage pathway. It is responsible for reforming IMP and GMP from hypoxanthine and guanine, respectively. In this manner purine bases are salvaged back into the purine nucleotide pool. Genetic defects that lead to the loss of HGPRT activity are the primary cause for which of the following conditions?
a. Gout
b. Lesch-Nyhan syndrome
c. Orotic aciduria
d. Severe combined immunodeficiency syndrome
e. Tay-Sachs disease

A

b. Lesch-Nyhan syndrome

132
Q

A 30-year-old white woman enters the emergency room complaining of nausea, severe abdominal pain, and prolonged constipation. She appeared distraught and was sweating. She described beginning an extremely low calorie diet within the past 2 months in an attempt to lose weight. Physical examination determined rapid heart rate, moderate hypertension, and weakness in the extremities. Also noted were mild dermatitis/blistering on her hands and scarring on her face.

You suspect porphyria. Which biochemical laboratory test(s) would be sufficient to determine the type of porphyria?
a. Urinary ALA and PBG.
b. Urinary and fecal PBG and porphyrins.
c. Porphyrin spectrofluorometry (plasma scan).
d. None of the above is sufficient alone.

A

c. Porphyrin spectrofluorometry (plasma scan).

133
Q

A 30-year-old white woman enters the emergency room complaining of nausea, severe abdominal pain, and prolonged constipation. She appeared distraught and was sweating. She described beginning an extremely low calorie diet within the past 2 months in an attempt to lose weight. Physical examination determined rapid heart rate, moderate hypertension, and weakness in the extremities. Also noted were mild dermatitis/blistering on her hands and scarring on her face.

Laboratory tests revealed elevated urinary PBG and coproporphyrin, and plasma fluorescence emission at 626 nm. Which type of porphyria does the patient have and what is the most likely biochemical explanation?
a. Acute intermittent porphyria; PBGD heterozygous enzyme deficiency causing PBG backlog.
b. Porphyria cutanea tarda; UROD homozygous enzyme deficiency causing uroporphyrin backlog.
c. Variegate porphyria; PPO heterozygous deficiency causing protoporphyrin IX backlog.
d. Variegate porphyria; PPO homozygous deficiency causing protoporphyrin IX backlog.

A

c. Variegate porphyria; PPO heterozygous deficiency causing protoporphyrin IX backlog.

134
Q

The second enzyme in the heme pathway, ALAD, is very sensitive to inhibition by heavy metals, for example, lead. Which of the following test results would distinguish lead-based poisoning from acute intermittent porphyria?
a. Decreased serum and urinary ALA and PBG
b. Increased serum and urinary ALA and PBG
c. Increased serum and urinary ALA, decreased serum and urinary PBG
d. Decreased serum and urinary ALA, increased serum and urinary PBG

A

c. Increased serum and urinary ALA, decreased serum and urinary PBG

135
Q

A 25-year old female sought treatment for her constant fatigue, lethargy, and depression. She was small in stature and had previously been diagnosed with attention-deficit disorder. On physical examination she was found to have an enlarged thyroid gland (goiter). Blood tests revealed elevated levels of T3, T4, and TSH, yet she did not exhibit typical symptoms of
hyperthyroidism. Which one of the following possibilities offers the most likely explanation of her symptoms?
a. Thyroid hormone overproduction because of a thyroid gland tumor
b. Hypersecretion of TSH because of a pituitary tumor
c. Genetic alteration in the thyroid receptor reducing its ability to bind thyroid hormone
d. Mutation in the TSH receptor in the thyroid gland reducing its ability to bind TSH
e. Iodide deficiency in the diet

A

c. Genetic alteration in the thyroid receptor reducing its ability to bind thyroid hormone

136
Q

In individuals with iodide deficiency, which one of the following is most likely?
a. TSH levels are elevated and directly stimulate growth of the thyroid gland to a very large size.
b. Mono- and diiodinated thyroid hormone molecules are produced, and elevated levels of these derivatives compensate for the deficiency.
c. TSH levels are decreased, relieving their inhibitory effects on thyroid cell proliferation.
d. Synthesis of the Na+, K+-ATPase is increased.

A

a. TSH levels are elevated and directly stimulate growth of the thyroid gland to a very large size.

137
Q

In women taking thyroid hormone replacement pills, the dosage must be adjusted if they start taking birth control pills. Which one of the following best explains this situation?
a. Thyroid hormones block the action of estrogens, so the estrogen dose must be increased.
b. Estrogens block the action of thyroid hormones, so the dose of thyroid hormone must be increased.
c. Progestins block the action of thyroid hormone, so the dose of thyroid hormone must be increased.
d. Thyroid hormones stimulate the action of estrogens, so the estrogen dose must be decreased.
e. Estrogens stimulate the action of thyroid hormone, so the dose of thyroid hormone must be decreased.

A

b. Estrogens block the action of thyroid hormones, so the dose of thyroid hormone must be increased.

138
Q

Following brain surgery involving transsphenoidal removal of a pituitary adenoma, a patient experienced polyuria, polydipsia, and nocturia. These symptoms appeared shortly after the surgery and had never been observed previously. Osmolarity of the urine was below normal, even if liquid consumption was restricted. Administration of desmopressin alleviated these symptoms. Which of the following possibilities is the most likely hypothesis to explain these symptoms?
a. Damage to the thirst mechanism from surgical trauma, leading to excessive consumption of liquids.
b. Damage to the pituitary or hypothalamus from surgical trauma, leading to decreased secretion of vasopressin.
c. Onset of diabetes mellitus following surgery
d. Renal injury
e. Oversecretion of angiotensin II following surgery

A

b. Damage to the pituitary or hypothalamus from surgical trauma, leading to decreased secretion of vasopressin.

139
Q

Desmopressin acts mainly by which of the following mechanisms?
a. Stimulating aldosterone secretion by the adrenal gland
b. Increasing synthesis of Na+ transporters in kidney distal tubule
c. Increasing aquaporin insertion into renal distal tubule apical membranes
d. Acting as an insulin sensitizer
e. Stimulation of angiotensin II release

A

c. Increasing aquaporin insertion into renal distal tubule apical membranes

140
Q

Which of the following would be least likely to stimulate vasopressin release from the posterior pituitary?
a. Dehydration
b. Stress
c. Angiotensin II
d. Atrial stretch receptors
e. Aldosterone

A

e. Aldosterone

141
Q

A 30-year-old female of normal weight was recently diagnosed with type II diabetes and hypertension. Menstrual cycles were irregular. In appearance she had unusually coarse features; a noticeable enlargement of the tongue, hands, and feet; and a deep voice. Although not pregnant or nursing, she unexpectedly began producing breast milk (galactorrhea). Which one of the following possibilities is most likely to explain all of these symptoms?
a. Hyperinsulinemia and insulin resistance
b. Pituitary tumor and growth hormone overproduction
c. Testosterone overproduction
d. Ovarian cysts
e. Transforming growth factor b overproduction

A

b. Pituitary tumor and growth hormone overproduction

142
Q

A 7-year-old child who was very small for his age began receiving treatment with growth hormone. Which one of the following metabolic alterations is most likely to be observed after beginning this treatment?
a. Inhibition of cartilage formation
b. Inhibition of gluconeogenesis
c. Inhibition of triglyceride breakdown and oxidation in adipocytes
d. Stimulation of IGF-1 secretion
e. Stimulation of protein breakdown

A

d. Stimulation of IGF-1 secretion

143
Q

The following polypeptide hormones each interact with receptors in the plasma membrane of their target cells. Which one triggers a signaling pathway that is directly stimulated by treatment of the cell with an inhibitor of cyclic AMP phosphodiesterase?
a. ACTH
b. Epidermal growth factor
c. Growth hormone
d. Insulin
e. Nerve growth factor

A

a. ACTH

144
Q

An obese 57-year-old woman did not yet exhibit symptoms of menopause but was diagnosed with polycystic ovary syndrome (PCOS) and insulin resistance. Her plasma levels of testosterone were above normal. Which one of the following is most likely in this case?
a. Hyperinsulinemia leading to androgen overproduction by the ovary and its conversion to estrogen in fat cells
b. Androgen overproduction by the adrenal gland and its conversion to estrogen in fat cells
c. Progesterone overproduction by the polycystic ovary leading to its conversion to estrogen
d. LH/FSH ratio = 1
e. Estrogen overproduction by the ovary and conversion to testosterone

A

a. Hyperinsulinemia leading to androgen overproduction by the ovary and its conversion to estrogen in fat cells

145
Q

Which one of the following changes is most likely to be observed in a postmenopausal woman who is not taking hormone supplementation?
a. Cessation of androgen secretion
b. Increased levels of FSH and LH
c. Increased osteoblast activity
d. Decreased levels of gonadotropin-releasing hormone
e. Increased progesterone levels

A

b. Increased levels of FSH and LH

146
Q

In a normal premenopausal woman, which one of the following is stimulated by progesterone?
a. Release of gonadotropin-releasing hormone by the pituitary
b. Ovulation
c. Development of the endometrium in preparation for possible pregnancy
d. Uterine contraction
e. Follicle development

A

c. Development of the endometrium in preparation for possible pregnancy

147
Q

A 29-year-old female patient exhibited a rounded face, hirsutism, upper body obesity, easily bruised skin, severe fatigue, muscle weakness, and anxiety. She also complained of irregular periods. A long- term asthma sufferer, she had been prescribed prednisone for the past 2 years. Findings on examination revealed high fasting blood glucose levels and high blood pressure. Cortisol levels were below normal. Which one of the following is the most likely explanation to account for the patient’s symptoms?
a. Decreased levels of insulin
b. Increased levels of testosterone
c. Decreased secretion of ACTH
d. Excess exogenous glucocorticoid hormone
e. Increased hepatic metabolism of steroid hormones

A

d. Excess exogenous glucocorticoid hormone

148
Q

A patient suffering from weakness, fatigue, nausea, and vomiting was found to have low blood concentrations of Na+ and Cl− and high levels of serum K+. Physical examination revealed a deep tanning of both exposed and unexposed parts of the body and dark pigmentation inside the mouth. The hyperpigmentation in this patient most likely resulted from which of the following?
a. Increased secretion of ACTH
b. Prolonged exposure of the patient to ultraviolet radiation
c. Excessive ingestion of b-carotene–containing foods
d. Activation of melanocytes caused by medication side effects
e. Inhibition of plasma membrane Na+, K+-ATPase

A

a. Increased secretion of ACTH

149
Q

Metyrapone is used to block the mitochondrial 11b-hydroxylaseinthe corticosteroid synthetic pathway and is administered to evaluate hypothalamic-pituitary-adrenal axis function. Which of the following results is most likely from this overnight diagnostic test in a healthy individual?
a. Feedback inhibition of cortisol biosynthesis
b. Increase in the levels of cortisol precursors
c. Decrease in ACTH levels
d. Inhibition of adenylate cyclase activity in adrenal cortical cells
e. Inhibition of pituitary function

A

b. Increase in the levels of cortisol precursors

150
Q

A 45-year-old female patient presents with hirsutism, striae, bruising, acne, and hyperpigmentation of the skin. After a thorough physical examination the physician notes that she also suffers from hypertension and shows signs of a “buffalo hump” on her back between the shoulders. Cushing syndrome is suspected and after laboratory tests show elevated blood cortisol levels she is given a dexamethasone suppression test. Her results are positive.

Following administration of dexamethasone, this patient exhibits elevated cortisol levels (a positive result) because of which of the following?
a. The humoral stress pathway can no longer regulate cortisol levels via a negative feedback loop.
b. There is a deficiency in the enzyme that breaks down dexamethasone, leading to excess amounts of glucocorticoid in the blood.
c. The anterior pituitary is nonresponsive to excess cortisol and is aberrantly producing excess CRH.
d. ACRH-secreting tumor of the adrenal glands is stimulating cortisol synthesis and is no longer responding to the negative feedback loop.

A

a. The humoral stress pathway can no longer regulate cortisol levels via a negative feedback loop.

151
Q

A 45-year-old female patient presents with hirsutism, striae, bruising, acne, and hyperpigmentation of the skin. After a thorough physical examination the physician notes that she also suffers from hypertension and shows signs of a “buffalo hump” on her back between the shoulders. Cushing syndrome is suspected and after laboratory tests show elevated blood cortisol levels she is given a dexamethasone suppression test. Her results are positive.

The hirsutism observed in the patient above is best explained by which of the following?
a. ACTH stimulating synthesis of adrenal androgens
b. CRH stimulating synthesis of adrenal androgens
c. Cortisol activating aldosterone receptors
d. Cortisol stimulating expression of adrenal androgen biosynthetic enzymes

A

a. ACTH stimulating synthesis of adrenal androgens

152
Q

The characteristic accumulation of adipose tissue in the facial, truncal, and cervical regions of the body in patients with Cushing syndrome is best explained by which of the following?
a. Excess adrenal androgens due to adrenal tumor
b. Excess cortisol over long periods of time
c. Cross stimulation of mineralocorticoid receptors by cortisol
d. Increased production of MSH
e. Increased proteolysis

A

b. Excess cortisol over long periods of time

153
Q

The most common cause of hypercortisolism is which of the following?
a. Adrenal tumor that secretes excess cortisol and mineralocorticoid hormones.
b. Lung tumor that secretes excess ACTH that leads to excess cortisol in the blood.
c. Administration of synthetic cortisol by physician
d. Pituitary adenoma that secretes excess ACTH, leading to excess cortisol in the blood.

A

d. Pituitary adenoma that secretes excess ACTH, leading to excess cortisol in the blood

154
Q

A 54-year-old patient complained of muscle weakness, fatigue, and depression. She had a recent episode of renal stones and a bone scan revealed osteopenia. She had not yet entered menopause. She has taken a daily multivitamin tablet plus an additional 500 mg of calcium citrate for the past 20 years. Results from blood chemistry analysis indicated elevated levels of serum calcium ion. Urinalysis indicated phosphaturia. The patient’s symptoms are most likely caused by which of the following?
a. Excessive intake of vitamin D
b. Excess synthesis of parathyroid hormone
c. Excessive intake of calcium
d. Excess synthesis of calcitonin
e. Excess intake of phosphate

A

b. Excess synthesis of parathyroid hormone

155
Q

Before the introduction of vitamin D-fortified milk, children who spent most of their time indoors often developed rickets. In these vitamin D-deficient children, the most likely explanation for their bone malformations is which of the following?
a. Excessive renal excretion of calcium
b. Excessive renal excretion of phosphate ion
c. Inadequate uptake of calcium in the intestine
d. Lack of weight-bearing exercise
e. Excessive renal tubular reabsorption of calcium

A

c. Inadequate uptake of calcium in the intestine

156
Q

Why can excessive ingestion of phosphate-containing softdrink in otherwise well-nourished individuals lead to decreased bone density?
a. Increased levels of blood phosphate ion interact with sensors on the membrane of parathyroid cells to stimulate parathyroid hormone release.
b. Phosphate ion binds the active site on calcium transporters in the intestine, inhibiting their ability to transport calcium.
c. Phosphate ion depletes cellular levels of ATP resulting in inhibition of Ca2+–ATPase calcium transporters.
D. Phosphate ion spontaneously forms an insoluble precipitate with calcium ion, decreasing its absorption in the intestine
e. Phosphate ion is excessively incorporated into bone, weakening its structure.

A

D. Phosphate ion spontaneously forms an insoluble precipitate with calcium ion, decreasing its absorption in the intestine

157
Q

A 6-month-old infant presents with an oral yeast infection and the patient’s history reveals that there have been persistent infections since approximately 1 month of age. Blood work reveals decreased number of T and B lymphocytes, and you thus suspect an immunodeficiency. Which of the following laboratory results would specifically implicate ADA deficiency as the molecular basis for the immunodeficiency seen in this patient?
a. Increased adenosine and deoxyadenosine in the blood
b. Elevated inosine and deoxyinosine in the blood
c. Decreased levels of adenosine and deoxyadenosine in the blood
d. Increased glucose levels in the blood
e. Increased creatinine levels in the blood

A

a. Increased adenosine and deoxyadenosine in the blood

158
Q

A 6-month-old infant was diagnosed with ADA-deficient SCID, which of the following treatment approaches is most likely to be successful?
a. Kidney transplantation
b. Liver transplantation together with immunoglobulin therapy
c. ADA enzyme replacement therapy
d. Immunoglobulin therapy
e. Dietary restriction of adenine

A

c. ADA enzyme replacement therapy

159
Q

Which of the following clinical parameters should be followed in the ADA-deficient 6-month- old infant following the use of ADA enzyme replacement therapy to assess the efficacy of the therapy?
a. Blood urea nitrogen
b. Hemoglobin levels
c. Erythrocyte cell counts
d. Immunoglobulin levels
e. Ferritin levels

A

d. Immunoglobulin levels

160
Q

A deficiency of ADA can lead to lymphopenia and immune dysfunction. The accumulation of which substance is most likely to result in these lymphotoxic effects?
a. Adenosine
b. Inosine
c. Deoxyadenosine
d. Uric acid
e. Hypoxanthine

A

c. Deoxyadenosine

161
Q

In addition to maple syrup urine disease, there are a few additional inborn errors of metabolism that also present with neonatal encephalopathy. These include hyperkeratosis syndromes such as B-ketothiolase deficiency, urea cycle defects, glycine encephalopathy, and propionic or methylmalonic academia. Which of the following tests would allow the definitive diagnosis of MSUD?
a. Urine protein analysis
b. Cranial Computed Tomography
c. CSF Culture
d. Plasma amino acid analysis

A

d. Plasma amino acid analysis

162
Q

Phe-Thr-Val-Tyr-Leu-Gly-Met→Phe-Thr-Val-Stop-TTT=ACA- GTT-TAT-CTC-GGG-ATG
a. Missense mutation
b. Nonsense mutation
c. Silent mutation
d. Repeat expansion

A

b. Nonsense mutation

163
Q

Which of the following is the most probable cause of the elevated ketone and low glucose readings?
a. Increased flux of acetyl CoA through the TCA cycle
b. Inadequate dietary intake
c. Increased protein mobilization
d. Increased pyruvate carboxylase activity

A

b. Inadequate dietary intake

164
Q

Alcohol metabolism is accomplished in various cellular compartments including the cytosol, mitochondria, and endoplasmic reticulum. Which reaction is least likely to inhibit the synthesis of glucose?
a. Ethanol → Acetaldehyde by cytosolic alcohol dehydrogenase
b. Ethanol → Acetaldehyde by the microsomal ethanol oxidizing system
c. Acetaldehyde → Acetic acid by mitochondrial acetaldehyde dehydrogenase
d. B-Hydroxybutyrate → Acetoacetate

A

b. Ethanol → Acetaldehyde by the microsomal ethanol oxidizing system

165
Q

A man who is apparently healthy complains of weakness and has a low erythrocyte count. The lady physician suspects a recent hemolytic crisis and checks his erythrocytes and glucose 6-phosphate dehydrogenase levels. His erythrocyte count is down and his glucose-6-phosphate dehydrogenase is normal. She knows that the pathway for protecting the erythrocyte membrane from damage is complex and that glutathione peroxidase is involved. Peroxidases use reducing equivalents produced by metabolism from glucose to reduce organic hydroperoxides to the hydroxyl level. Which of the following statements best describes the action of glutathione peroxidase?
a. Glutathione peroxidase reduces glutathione disulfide back to reduced glutathione
b. Glutathione peroxidase produces 1 molecule of CO, in its reaction cycle
c. Glutathione peroxidase requires a selenium ion cofactor
d. Glutathione peroxidase reduces organic hydroperoxides to organic hydroxyl compounds
e. Glutathione peroxidase can use NADPH or NADH directly as a source of reducing equivalents

A

d. Glutathione peroxidase reduces organic hydroperoxides to organic hydroxyl compounds

166
Q

A lady physician, while on her hematology rounds, is assigned to male patient who has had an episode of hemolysis. Based on his recent consumption of fava bean soup and his Middle Eastern heritage, she suspects a fava bean induced episode of active oxygen radicals may have caused the hemolysis. She knows that fava beans contain divicine, isouramil, and aglycones that are redox active, permitting the transfer of electrons from one compound to another. When oxygen radical species are formed in individuals with deficient glucose 6-phosphate dehydrogenase activity, membrane lipids can undergo hydroperoxide formation. Which of the following statements is least accurate about erythrocytes in this patient?
a. Red blood cell membrane elasticity is reduced.
b. Red blood cells are more easily hemolyzed.
c. Red blood cells are more permeable.
d. Red blood cells easily traverse capillary beds.
e. Red blood cells are less able to reduce fatty acyl hydroperoxides to hydroxyl forms.

A

d. Red blood cells easily traverse capillary beds.

167
Q

A 40-year-old man presents to his primary care physician due to a painful lesion on his scrotum. On examination, the physician observed what appeared to be herpetic vesicles on the left underside of the scrotum. He prescribed a course of valacyclovir. Valacyclovir treats outbreaks of HSV infection by which of the following?
A. Acting as a protease inhibitor
B. Inhibiting protein translation
C. Inhibiting reverse transcriptase
D. Inhibiting packaging of new viral particles
E. Inhibiting replication of the viral genome

A

E. Inhibiting replication of the viral genome

168
Q

One hallmark of MSUD is the presence of ketones in the urine. A quick screen involves the addition of dinitrophenylhydrazine (2-DNP) to the patient’s urine sample. A positive result is indicated by the formation of a yellow-white precipitate. The type of reaction responsible for this product can best be described as which of the following?
A. Isomerization reaction
B. Condensation reaction
C. Combustion reaction
D. Acid/base reaction

A

B. Condensation reaction

169
Q

A healthy couple of Old World Mennonite descent is preparing to have a child. Given that the carrier incidence of the trait is 1 in 10, what is the probability that their child will have MSUD?
A. 1 in 10
B. 1 in 100
C. 1 in 400
D. 1 in 200

A

C. 1 in 400

170
Q

The disorientation observed in the patient is most likely caused by which of the following?
A. Alcohol damage to the kidney
B. Decreased lung surfactant efficacy caused by excessive alcohol intake
C. Increased ketone bodies stimulate diuresis
D. Mentation is suppressed by elevated ketone bodies
E. Neuronal and glial metabolism is reduced

A

E. Neuronal and glial metabolism is reduced

171
Q

The patient is treated by IV thiamine and glucose solutions. Although the patient appears to be improving, ketone test strip analysis indicates an increase in urinary ketones. This is best explained by which of the following?
A. Conversion of β-hydroxybutyrate to acetoacetate
B. Decreased activity of the TCA cycle
C. Increased catabolism of ketogenic amino acids
D. Inhibition of pyruvate dehydrogenase
E. Conversion of lactate to pyruvate

A

A. Conversion of β-hydroxybutyrate to acetoacetate

172
Q

A 64-year-old man is presented to his family doctor with complaints of frequent episodes of dizziness and of numbness in his legs. During a routine history and physical examination, the doctor finds that the patient leads a sedentary lifestyle, is obese (body mass index of 32 kg/m2), and has hypertension (blood pressure of 200/120 mm Hg). The patient is asked to return to the clinic 1 week later in the fasting state, during which time a blood specimen is obtained, and a glucose tolerance test is performed. Humoral analysis reveals fasting hyperglycemia, hyperinsulinemia, dyslipidemia, and glucose intolerance. The diagnosis is type 2 diabetes mellitus. Alterations in substrate metabolism within which of the following organs can be a cause for the observed humoral analysis?
A. Brain
B. Kidney
C. Liver
D. Heart
E. Spleen

A

C. Liver

173
Q

A mutation, leading to decreased activity, in the gene encoding for which of these proteins is most consistent with this clinical presentation?
A. Glucagon
B. Glucose transporter isoform 1
C. GP
D. Pyruvate carboxylase
E. PP1

A

E. PP1

174
Q

Which of the following complications is less likely to occur in people with type 2 diabetes compared with those with type 1 diabetes?
A. Retinopathy
B. Weight gain
C. Cardiovascular disease
D. Hypoglycemic coma
E. Neuropathy

A

D. Hypoglycemic coma

175
Q

A patient in your care has had a hemolytic crisis event and you have on your differential diagnosis notes that the event may indicate a dysfunction of G6PDH. You know that G6PDH in the red blood cell uses glucose 6-phosphate from the glycolytic pathway to form NADPH through a series of reactions. A person with deficient G6PDH would be expected to have all of the following characteristics except which of the following?
A. Increased proportion of glucose metabolism to pyruvate versus ribulose 5-phosphate than in normal
B. Decreased glutathione reductase activity
C. Decreased glutathione peroxidase activity
D. Increased reduction of organic hydroperoxides to hydroxyl products
E. Decreased production of gluconolactone-6-phosphate

A

D. Increased reduction of organic hydroperoxides to hydroxyl products

176
Q

On the third day of life, a female neonate born of nonconsanguineous parents began to vomit and became lethargic and hypotonic. Laboratory analysis indicated that she was hypoglycemic, acidotic (pH 7.3), and hyperammonemic. The patient remained lethargic, hypotonic, and became unresponsive despite administration of intravenous glucose. Further laboratory analyses revealed lactic acidosis, elevated ketones, and increased levels of the organic acids butyrate, adipate, and ethylmalonate in the urine. The neonate most likely is deficient in which of the following?
A. Carnitine palmitoyltransferase I
B. Carnitine acylcarnitine translocase
C. MCAD
D. SCAD
E. Succinate dehydrogenase

A

D. SCAD

177
Q

A 5-year-old boy had several unexplained episodes of lethargy and coma associated with fasting, hypoglycemia with low ketones, and dicarboxylic aciduria beginning at about 18 months of age. Hospitalization following a recurrence at 5 years led to the diagnosis of a deficiency of VLCAD. Which of the following would be contraindicated in the management of this patient?
A. Diet high in carbohydrates
B. Carnitine supplementation
C. High fat, ketogenic diet
D. Frequent feeding
E. Medium-chain triglyceride supplementation

A

C. High fat, ketogenic diet

178
Q

The first step in the β-oxidation of fatty acyl-CoA is catalyzed by a family of enzymes, each of which are specific for different chain length fatty acyl groups. The activity of this family of enzymes would be most negatively affected by a dietary deficiency in which of the following vitamins?
A. Ascorbic acid
B. Biotin
C. Niacin
D. Riboflavin
E. Thiamine

A

D. Riboflavin

179
Q

Catecholamine secretion from a pheochromocytoma is considered unregulated. This is because the tumor:
A. Lacks β-adrenergic receptors
B. Contains defective transport molecules
C. Is not innervated
D. Lacks α-adrenergic receptors

A

C. Is not innervated

180
Q

Which of the following is not a tissue response of increased catecholamine secretion?
A. Constriction of airways
B. Hepatic conversion of glycogen to glucose
C. Increased heart rate
D. Dilation of blood vessels of skeletal muscle

A

A. Constriction of airways

181
Q

Despite recommended preoperative preparation with α- and β-adrenergic blockers, severe hemodynamic instability may occur during operations to resect pheochromocytoma. Some preoperative protocols recommend the addition of the tyrosine hydroxylase inhibitor, metyrosine, in an attempt to better manage the hypertension of patients with pheochromocytoma under-going surgical resection. What mechanism does this treatment utilize to control hemodynamic stability?
A. Catecholamine metabolism
B. Catecholamine synthesis
C. Catecholamine-receptor binding
D. Catecholamine secretion

A

B. Catecholamine synthesis

182
Q

When oxygen is bound to hemoglobin, the ferrous ion (Fe2+) can be oxidized to the ferric ion (Fe3+), which no longer binds oxygen. This results in production of the superoxide anion radical and methemoglobin. Methemoglobin levels are normally kept at a minimum by the action of which enzyme?
A. Glucose 6-phosphate dehydrogenase
B. Malic enzyme
C. NADH-cytochrome b5 reductase
D. NADH dehydrogenase
E. NADPH-cytochrome b5 reductase

A

C. NADH-cytochrome b5 reductase

183
Q

A young Caucasian girl is brought to the pediatric clinic by her parents because they are concerned that she has a bluish hue to her skin. The physician suspects that the child has a congenital form of methemoglobinemia. However, an extract of her blood is able to convert added methemoglobin to hemoglobin. It is likely that the girl has which abnormal hemoglobin variant?
A. Hemoglobin A1C
B. Hemoglobin C
C. Hemoglobin F
D. Hemoglobin M
E. Hemoglobin S

A

D. Hemoglobin M

184
Q

A 2-month-old male infant presents to the emergency department following a 2-day bout with diarrhea and poor oral food intake. The infant is lethargic, has a dusky color, and signs of dehydration. Lungs are clear to auscultation but his breathing is labored. In addition to dehydration, the child is diagnosed with methemoglobinemia. To treat the methemoglobinemia, the child is administered methylene blue. Methylene blue reduces methemoglobin by utilizing reducing equivalents (electrons) drawn from which of the following?
A. Tricarboxylic acid cycle
B. Pentose phosphate pathway
C. Oxidation of glutamate to α-ketoglutarate
D. Glycolysis
E. β-Oxidation pathway

A

B. Pentose phosphate pathway

185
Q

After an overnight fast, the blood levels of which of the following will be higher to a 25 years old female who has carnitine deficiency than to an apparently normal person?
a. Acetoacetate
b. Fatty acids
c. 3-Hydroxybutyrate
d. Glucose

A

b. Fatty acids

186
Q

Gene therapy
a. Use gene technology on humans to treat a disease
b. Alter the characteristics of organisms
c. Use genetically modified organisms to produce chemicals
d. Medical or industrial applications

A

a. Use gene technology on humans to treat a disease

187
Q

After fasting for 12 hours, you consume 6 pieces of doughnut from Mister Donut. This meal will
a. Increase blood glucagon levels
b. Increase the rate of gluconeogenesis
c. Replenish liver glycogen stores
d. Reduce the rate at which fatty acids are converted to adipose triacylglycerols

A

c. Replenish liver glycogen stores

188
Q

Which one of the following is a characteristic of phospholipids?
a. Are a major component of membranes
b. Always contain choline and glycerol
c. Are an important source of energy during fasting
d. Are not charged in the body

A

a. Are a major component of membranes

189
Q

If the enzyme concentration for a biochemical reaction is increased 100-fold, the equilibrium constant for the reaction will
a. Increase in proportion to the enzyme concentration
b. Decrease twofold
c. Remain the same
d. Change inversely with the enzyme concentration

A

c. Remain the same

190
Q

Single disease-causing mutation affects multiple organ
a. Locus heterogeneity
b. Anticipation
c. Pleiotropy
d. Sex-linked disorders

A

c. Pleiotropy

191
Q

In the pathway for triacylglycerol synthesis in the liver
a. Phosphatidic acid is an intermediate
b. Fatty acids react with glycerol 3-phosphate
c. Coenzyme A is not required
d. A 2-monoacylglycerol is an intermediate

A

a. Phosphatidic acid is an intermediate

192
Q

If you are resting after an overnight fast, which of the following would be observed?
a. Liver gluconeogenesis is not an important process
b. Muscle glycogen stores are used to maintain blood glucose
c. Liver glycogen stores are completely depleted
d. Fatty acids are released from adipose triacylglycerol stores

A

d. Fatty acids are released from adipose triacylglycerol stores

193
Q

True to heme
a. Colourless
b. Intermediates between porphobilinogen and protoporphyrin in heme synthesis
c. Chemically reduced form
d. Porphyrin-ring structure constructed from 4 pyrrole rings

A

d. Porphyrin-ring structure constructed from 4 pyrrole rings

194
Q

How many net molecules of ATP are generated in the conversion of glucose to pyruvate?
a. 1
b. 2
c. 3
d. 0

A

b. 2

195
Q

A competitive inhibitor of an enzyme
a. Decreases Vmax but does not affect Km
b. Increases Km but does not affect Vmax
c. Decreases Km but does not affect Vmax
d. Increases Vmax but does not affect Km

A

b. Increases Km but does not affect Vmax

196
Q

25-year-old female who is with hyperlipoproteinemia would be most likely to benefit from a low-carbohydrate diet if the lipoproteins that are elevated in the blood are
a. VLDL
b. HDL
c. Chylomicrons
d. LDL

A

a. VLDL

197
Q

Tetracycline, streptomycin, and erythromycin are effective antibiotics because they inhibit
a. RNA synthesis in eukaryotes
b. RNA synthesis in prokaryotes
c. Protein synthesis on cytoplasmic ribosomes of eukaryotes
d. Protein synthesis in prokaryotes

A

c. Protein synthesis on cytoplasmic ribosomes of eukaryotes

198
Q

The parent purine nucleotide
a. Cytosine monophosphate
b. Guanosine monophosphate
c. Inosine monophosphate
d. Adenosine monophosphate

A

c. Inosine monophosphate

199
Q

In which compartment of the cell does glycolysis occur?
a. Rough endoplasmic reticulum
b. Soluble cytoplasm
c. Nucleus
d. Mitochondrion

A

b. Soluble cytoplasm

200
Q

An infant with an enlarged liver has a glucose 6-phosphatase deficiency. This infant
a. Can convert both alanine and glycerol to glucose to maintain blood glucose levels
b. Cannot maintain blood glucose levels either by glycogenolysis or by gluconeogenesis
c. Can use muscle glycogen to maintain blood glucose levels
d. Can use liver glycogen to maintain blood glucose levels

A

b. Cannot maintain blood glucose levels either by glycogenolysis or by gluconeogenesis

201
Q

You see a female patient in a follow-up after discharge from the hospital. She was treated for ketoacidosis and hyperglycemia and now is on basal and rapid acting insulins. You wonder if she really has type 1 diabetes mellitus and was in ketoacidosis or has type 2 diabetes mellitus and had a hyperosmolar state with lactic acidosis. Which of the following laboratory tests would help you determine whether this patient has type 1 or type 2 diabetes mellitus?
a. Hemoglobin A1C levels
b. Insulin levels
c. C-peptide levels
d. Fasting blood glucose levels

A

c. C-peptide levels

202
Q

Pyrimidine exclusive to RNA
a. Adenine
b. Uracil
c. Guanine
d. Cytosine
e. Thymine

A

b. Uracil

203
Q

Dietary fructose is phosphorylated in the liver and cleaved to form
a. One molecule each of dihydroxyacetone phosphate and glyceraldehyde 3-phosphate
b. One molecule each of dihydroxyacetone phosphate and glyceraldehyde
c. One molecule each of dihydroxyacetone and glyceraldehyde 3-phosphate
d. Two molecules of dihydroxyacetone phosphate
e. Two molecules of glyceraldehyde 3-phosphate

A

b. One molecule each of dihydroxyacetone phosphate and glyceraldehyde

204
Q

Which of the following amino acids that disrupt an alpha helix contains a bulky side chain?
a. Histidine
b. Proline
c. Aspartate
d. Tryptophan

A

d. Tryptophan

205
Q

Analysis of a cell line that rapidly transforms into a tumor cell line demonstrated an increased mutation rate within the cell. Further analysis indicated that there was a mutation in the DNA polymerase enzyme that synthesizes the leading strand. This inactivating mutation is likely to be in which of the following activities of this DNA polymerase?
a. Phosphodiester bond making capability
b. 3′-5′ exonuclease activity
c. 5′-3′ exonuclease activity
d. Uracil-DNA glycosylase activity

A

b. 3′-5′ exonuclease activity

206
Q

Cytochrome P450 enzymes are predominantly found in which of the following organs?
a. Brain
b. Liver
c. Intestine
d. Kidneys

A

b. Liver

207
Q

The following statements regarding use of the Henderson-Hasselbalch equation, the correct statement is that it can be used to
a. Calculate the degree of ionization of a weak acid with a known pKa value in a solution of known pH
b. Prepare a solution of lactic acid (pKa = 3.86) and sodium lactate with a pH of 7.5
c. Calculate the pH of a solution containing a weak acid if the pKa value for that acid is known
d. Calculate the pH of a solution at the end point of a titration of a weak acid with a strong base

A

c. Calculate the pH of a solution containing a weak acid if the pKa value for that acid is known

208
Q

Approximately how many net moles of ATP are generated when one mole of beta-hydroxybutyrate is oxidized to carbon dioxide and water in skeletal muscle?
a. 26
b. 25
c. 24
d. 23

A

a. 26

209
Q

A female molecular biologist is replicating human DNA in a test tube and has added intact DNA, the replisome complex, and the four deoxyribonucleoside triphosphates. To her surprise, there was no DNA synthesized, as determined by the incorporation of radio-labeled precursors into acid-precipitable material. Her failure to synthesize DNA is most likely due to a lack of which of the following in her reaction mixture?
a. Dideoxynucleoside triphosphates
b. Reverse transcriptase
c. Sigma factor
d. Ribonucleoside triphosphates

A

c. Sigma factor

210
Q

Which of the following statements concerning replication of DNA is true?
a. It requires a DNA template that is copied in its 5’ to 3’ direction
b. It produces one newly synthesized double helix and one composed of the two parental strands
c. It progresses in both directions away from each point of origin on the chromosome

A

c. It progresses in both directions away from each point of origin on the chromosome

211
Q

30 years old female had new glasses prescribed by her optometrist. Less than a week later, her prescription was inadequate and she could not see well with her new glasses. Her optometrist checked her vision twice more over the next week and her prescription was different both times. Her optometrist referred her to an ophthalmologist. What is the reason she is having such rapid changes in her glasses prescription?
a. Elevated levels of galactose in the lens
b. Elevated levels of glucose in the lens
c. Cataract formation
d. Elevated levels of sorbitol in the lens

A

b. Elevated levels of glucose in the lens

212
Q

Being a student, when compared to your state after an overnight fast, a person who fasts for 1 week will have
a. More adipose tissue
b. Lower levels of ketone bodies in the blood
c. Higher levels of blood glucose
d. Less muscle protein

A

d. Less muscle protein

213
Q

The resident-on-duty administered phenobarbital. A day after, the patient complains of myalgia, headache, abdominal cramping, paresthesia and other manifestation suggestive of severe lead toxicity which was confirmed by laboratory testing. The resident-on-duty made an investigation and found out that the drug, which was produced in China, was heavily contaminated with lead. Which of the enzymes below is extremely sensitive to inhibition by heavy metal ions?
a. Delta-Aminolevulinic acid dehydratase
b. Delta-Aminolevulinic acid synthase
c. Coproporphyrinogen oxidase
d. Protoporphyrinogen oxidase

A

a. Delta-Aminolevulinic acid dehydratase

214
Q

Which of the following is made of several enzymes with helicase and nuclease activity?
a. Micro-RNAs
b. RNA-induced silencing complex
c. Short interfering RNA
d. RNAi

A

c. Short interfering RNA

215
Q

If the population is small, random effects, such as increased fertility or survival of the carriers of a mutation, occurring for reasons unrelated to carrying the mutant allele, may cause the allele frequency to change from one generation to the next
a. Genetic drift
b. Gene flow
c. Effects of consanguinity

A

a. Genetic drift

216
Q

A nucleoside
a. Made of a pentose sugar and a base
b. Made of a pentose sugar, a base, and one phosphate group
c. Made of a pentose sugar, a base, and three phosphate groups
d. Made of a pentose sugar, a base, and two phosphate groups

A

a. Made of a pentose sugar and a base

217
Q

Which one of the following statements about fatty acids is TRUE?
a. Fatty acids are very soluble in water and need no carrier in the blood
b. Fatty acids may be oxidized to CO2 and H2O in the mitochondria of red blood cells
c. When fatty acids are activated in the cytosol, ATP is converted to ADP
d. Fatty acyl groups are covalently linked to carnitine by an enzyme inhibited by malonyl CoA

A

d. Fatty acyl groups are covalently linked to carnitine by an enzyme inhibited by malonyl CoA

218
Q

Regarding the mechanism of the Bohr effect, which of the following interactions leads to a greater affinity of deoxy form of hemoglobin for protons (than oxyhemoglobin) which then decreases oxygen affinity
a. Hydrogen Bonds
b. Ionic bonds
c. Hydrophobic interactions
d. Disulfide bonds

A

b. Ionic bonds

219
Q

A male patient was discharged from the hospital improved. He hates hospital food so he voraciously allowed himself to enjoy the pleasure of eating organ meats at home. Dietary purines are converted to uric acid in which of the following locations?
a. The intestinal mucosal cells which are for absorption
b. The liver which is the main metabolic organ
c. The kidneys which are the main organs of excretion
d. The intestinal flora

A

a. The intestinal mucosal cells which are for absorption

220
Q

The folate analogue which selectively inhibit bacterial dihydrofolate reductase
a. Methotrexate
b. Sulfonamides
c. Thioguanine
d. 5-Fluorouracil

A

a. Methotrexate

221
Q

Which B complex vitamin is the precursor of coenzyme A?
a. Pantothenate
b. Riboflavin
c. Nicotinamide
d. Niacin

A

a. Pantothenate

222
Q

Which of the following reagents of the Sanger dideoxy method of DNA sequencing is from a clinical sample?
a. 4 standard deoxynucleotide bases
b. Single-stranded DNA fragment
c. 2’, 3’-Dideoxynucleotide triphosphates
d. DNA polymerase

A

c. 2’, 3’-Dideoxynucleotide triphosphates

223
Q

The purine nucleotide cycle
a. Enzyme deficiencies lead to hypoglycemia, lactic acidosis and muscle fatigue during exercise
b. Net effect is deamination of aspartate to malate then to fumarate
c. Important in the liver and muscle
d. Anaplerotically to replenish TCA cycle intermediates

A

a. Enzyme deficiencies lead to hypoglycemia, lactic acidosis and muscle fatigue during exercise

224
Q

A 25-year-old female suffers with an LDL-receptor deficiency and was treated with lovastatin. As a consequence of the action of this drug, she should have
a. Increased ACAT activity
b. Increased de novo cholesterol synthesis
c. Fewer LDL receptors in cell membranes
d. Lower blood cholesterol levels

A

d. Lower blood cholesterol levels

225
Q

When the pH for a solution of this acid is equal to the pK, the ratio of the concentrations of the salt and the acid ([A-]/[HA]) is?
a. 2
b. 3
c. 0
d. 1

A

d. 1

226
Q

Cancer cells may contain hundreds of extra copies of proto-oncogenes
a. Viral stimulation
b. Chromosomal translocation
c. Amplification
d. Chromosomal translocation into a transcriptionally active region

A

c. Amplification

227
Q

Which of the following statements concerning Okazaki fragments is TRUE?
a. They are synthesized on the leading strand during replication
b. They are relatively short polydeoxyribonucleotides with a few ribonucleotide residues at the 5’ end
c. They are regions of DNA that do not code for the amino acids in a protein
d. They are produced by restriction enzymes

A

d. They are produced by restriction enzymes

228
Q

A 25 years old female, who has a deficiency of muscle phosphorylase, was examined while exercising her forearm by squeezing a rubber ball. Compared to a normal person performing the same exercise, this patient
a. Had increased glucose levels in blood drawn from her forearm
b. Had lower levels of glycogen in biopsies of her forearm muscle
c. Could exercise for a longer period of time without fatigue
d. Had decreased lactate levels in blood drawn from her forearm

A

d. Had decreased lactate levels in blood drawn from her forearm

229
Q

The closer the relationship the greater the likelihood of production of a child with the genetic disease
a. Genetic drift
b. Effects of consanguinity
c. Gene flow
d. New mutation

A

a. Genetic drift

230
Q

A 25 years old female has a genetic defect that causes intestinal epithelial cells to produce disaccharidases of much lower activity than normal. Compared to a normal person, after eating a bowl of milk and oatmeal, this patient will have higher levels of
a. Glycogen in the muscles
b. Maltose, sucrose, and lactose in the stool
c. Galactose and fructose in the blood
d. Starch in the stool

A

b. Maltose, sucrose, and lactose in the stool

231
Q

The peptide bond has a backbone of atoms in which of the following sequences
a. C-N-N-C
b. N-C-C-C
c. C-C-C-N
d. C-C-N-C

A

d. C-C-N-C

232
Q

In gluconeogenesis, both alanine and lactate are converted in a single step to
a. Phosphoenolpyruvate
b. Pyruvate
c. Acetyl CoA
d. Oxaloacetate

A

b. Pyruvate

233
Q

A 30-year-old man works as an administrative assistant for a large company, opened a package and found a suspicious white powder. Analysis of the powder indicates that it contained traces of the bacterium Bacillus anthracis. He was treated with ciprofloxacin, an effective antibiotic. Ciprofloxacin’s mechanism of action is best described as an inhibition of which of the following?
a. DNA gyrase
b. Bacterial peptidyl transferase activity
c. Bacterial dihydrofolate reductase
d. Bacterial RNA polymerase

A

a. DNA gyrase

234
Q

The reactions of the TCA cycle oxidizing succinate to oxaloacetate
a. Produce one high-energy phosphate bond
b. Require both NAD+ and FAD
c. Require coenzyme A
d. Include an isomerization reaction

A

b. Require both NAD+ and FAD

235
Q

A lady physician sees a very sick patient (vomiting and bloody diarrhea, dehydration, and mental status changes) in the emergency department, who, as she was told, was an amateur chef trying out a new creation in which he wanted to experiment with the extracts of castor beans. This person’s symptoms are all due to which of the following?
a. Ribosomal inactivation by covalent modification
b. Inhibition of amino-acyl tRNA synthetases
c. Inhibition of RNA polymerase I
d. Inhibition of RNA polymerase II

A

a. Ribosomal inactivation by covalent modification

236
Q

The sequence of part of a DNA strand is - ATTCGATTGCCCACGT-. When this strand is used as a template for DNA synthesis, the product will be which one of the following?
a. TAAGCTAACGGGTGCA
b. ACGUGGGCAAUCGAAU
c. UAAGCUAACGGGUGCA
d. ACGTGGGCAATCGAAT

A

a. TAAGCTAACGGGTGCA

237
Q

Main cellular processes and functions are suspended
a. Apoptosis
b. Cell division
c. Cancer formation
d. Senescence

A

d. Senescence

238
Q

A 30 years old man is suffering from chills, vomiting, and cramping and was rushed to the emergency department. He had eaten wild mushrooms for dinner that he had picked earlier in the day. His symptoms are due to an inhibition of which of the following enzymes?
a. Telomerase
b. RNA polymerase II
c. RNA polymerase I
d. RNA polymerase III

A

b. RNA polymerase II

239
Q

Which of the following steps in the conversion of genetic information into a final processed gene product is the most important mode for control of eukaryotic gene expression?
a. Initiation of transcription
b. Transport of transcript to cytoplasm
c. Transcript processing
d. Translation

A

a. Initiation of transcription

240
Q

After digestion of a piece of cake that contains flour, milk, and sucrose as its primary ingredients, the major carbohydrate products entering the blood are;
a. Glucose
b. Galactose and glucose
c. Fructose and galactose
d. Glucose, fructose, and galactose

A

d. Glucose, fructose, and galactose

241
Q

Which of the following statements about protein structure is correct?
a. The protomers of polymeric proteins are linked by covalent bonds
b. The stability of the alpha helix is mainly due to hydrophobic interactions
c. Globular proteins tend to fold into configurations that keep hydrophobic side chains in the interior of the molecule
d. The extended beta-configuration is not found in globular proteins

A

c. Globular proteins tend to fold into configurations that keep hydrophobic side chains in the interior of the molecule

242
Q

Which one of the following is a characteristic of the product of the fatty acid synthase complex in the liver?
a. May be elongated to stearic acid
b. May be reduced to form oleic acid
c. May be oxidized directly to palmitic acid
d. May be converted to arachidonic acid

A

a. May be elongated to stearic acid

243
Q

A 30 years old man is a Thalassemia minor patient. An electrophoretic analysis of Hb level in the indicates that, although there is a decrease in the relative amount of the β- chain with respect to the α-chain, both the β- and the α-chains migrate at the same position as normal chains. His anemia is most likely caused by which of the following?
a. Mutation in the promoter of the β-chain gene
b. Defect in an enzyme involved in heme synthesis
c. Frameshift mutation in the coding region of the gene coding for the β-chain
d. Point mutation in the coding region of the gene coding for the β-chain

A

a. Mutation in the promoter of the β-chain gene

244
Q

A man with Parkinson disease fell to the ground hitting his head to the concrete pavement. He was rushed to the emergency room where he had an episode of tonic clonic seizures. If the resident-on-duty administers phenobarbital, which of the enzymes below will be increased in response to the drug because of increased cytochrome P450 synthesis and enhanced heme consumption
a. delta-Aminolevulinic acid synthase
b. Coproporphyrinogen oxidase
c. Protoporphyrinogen oxidase
d. delta-Aminolevulinic acid dehydratase

A

c. Protoporphyrinogen oxidase

245
Q

Which one of the following is a characteristic of prostaglandins?
a. Are synthesized from polyunsaturated fatty acids
b. Are derived from fatty acids with 22 carbons
c. Do not contain keto or hydroxy groups
d. Contain ring structures with 8 carbons

A

a. Are synthesized from polyunsaturated fatty acids

246
Q

Erythromycin
a. The drug binds to the 50S ribosomal subunit of bacteria and blocks translocation
b. The drug binds to the 30S ribosomal subunit of bacteria and blocks peptide bond formation
c. The drug binds to the 50S ribosomal subunit of bacteria and inhibits f-met-tRNAi binding
d. The drug binds to the 30S ribosomal subunit of bacteria and blocks initiation of protein synthesis

A

a. The drug binds to the 50S ribosomal subunit of bacteria and blocks translocation

247
Q

Which chromosome has the most genes?
a. 3
b. 1
c. 2
d. 4

A

b. 1

248
Q

Therapeutic levels are not reached
a. Major allelic variant of P450 genes - Poor metabolism
b. Major allelic variant of P450 genes - Ultrarapid metabolism
c. Major allelic variant of P450 genes - Extensive metabolism
d. Major allelic variant of P450 genes - Intermediate metabolism

A

a. Major allelic variant of P450 genes - Poor metabolism

249
Q

A 25-year-old female presents to the emergency department after ingesting an insecticide. Her respiration rate is very low. Information indicates that this particular insecticide binds to and completely inhibits cytochrome C. Therefore, in this woman’s mitochondria
a. Cytochromes a and a3 would be in the reduced state
b. The rate of ATP synthesis would be approximately zero
c. Coenzyme Q would be in the oxidized state
d. The rate of CO2 production would be increased

A

b. The rate of ATP synthesis would be approximately zero

250
Q

Which of the following applications of polymerase chain reaction detects the actual presence of the virus by amplifying viral DNA, even in the absence of the host’s antibody response?
a. Forensic analysis of DNA samples
b. Prenatal diagnosis and carrier detection of hereditary diseases
c. Detection of low-abundance nucleic acid sequences
d. Comparison of normal cloned gene with an uncloned mutant form of the gene

A

c. Detection of low-abundance nucleic acid sequences

251
Q

A 25 years old female who suffers with type IIA hyperlipoproteinemia had a blood cholesterol level of 360 mg/dl (recommended level below 200 mg/dl) and blood triglyceride (triacylglycerol) levels of 140 mg/dl (recommended level below 160 mg/dl). She most likely has
a. Decreased ability for receptor-mediated endocytosis of LDL
b. Decreased ability to convert VLDL to IDL
c. Decreased ability to degrade the triacylglycerols of chylomicrons
d. Elevation of HDL in the blood
e. Increased ability to produce VLDL

A

a. Decreased ability for receptor-mediated endocytosis of LDL

252
Q

Major DNA repair systems
a. Direct repair
b. Recombination repair
c. Repair of single-strand damage
d. All of the above

A

b. Recombination repair

253
Q

Which of the following do not belong to the group?
a. VLDL
b. Beta lipoproteins
c. HDL
d. VHDL

A

d. VHDL

254
Q

A 25 years old female, a pregnant woman who has a lactase deficiency and cannot tolerate milk in her diet, is concerned that she will not be able to produce milk of sufficient caloric value to nourish her baby. She should be advised that
a. She can produce lactose by degrading alpha-lactalbumin
b. She will not be able to breast-feed her baby because she cannot produce lactose
c. She must eat pure galactose in order to produce the galactose moiety of lactose
d. The production of lactose by the mammary gland does not require the ingestion of milk or milk products

A

d. The production of lactose by the mammary gland does not require the ingestion of milk or milk products

255
Q

If a 25 years old female is suffering from a deficiency in the enzyme acetyl CoA carboxylase, what metabolite/s is likely to accumulate in her serum and urine?
a. Ketone bodies
b. Long chain fatty acids
c. Short chain fatty acids
d. Malonyl CoA

A

b. Long chain fatty acids

256
Q

Which one of the following occurs during beta-oxidation?
a. Carbon 2 of the fatty acid is oxidized to form a beta-hydroxy compound
b. NAD+ removes water from the beta-hydroxy fatty acyl CoA intermediate
c. Thiolase removes one carbon from the beta-keto intermediate
d. FAD is required to form a double bond in fatty acyl CoA
e. Two acetyl CoA molecules are produced in each turn of the beta-oxidation spiral

A

d. FAD is required to form a double bond in fatty acyl CoA

257
Q

Which of the following amino acids binds directly to Fe++ in myoglobin?
a. Tyrosine
b. Aspartate
c. Arginine
d. Histidine

A

d. Histidine

258
Q

A patient has a dominant disorder but it does not manifest clinically
a. Variable expression
b. New mutation
c. Incomplete penetrance
d. Founder effect

A

c. Incomplete penetrance

259
Q

The most common trisomy
a. Turner’s syndrome
b. Klinefelter’s syndrome
c. Edward syndrome
d. Down’s syndrome

A

d. Down’s syndrome

260
Q

Amino acid precursor of histamine
a. Histidine
b. Glutamate
c. Tyrosine
d. Tryptophan

A

a. Histidine

261
Q

Patients with xeroderma pigmentosum suffer DNA damage when they are exposed to UV light because UV light causes the formation of;
a. Anhydride bonds between phosphate groups in DNA
b. Pyrimidine dimers in DNA
c. Deoxyribose dimers in DNA
d. Purine dimers in DNA

A

b. Pyrimidine dimers in DNA

262
Q

Assuming that you passed biochemistry written revalida, became a licensed physician, and later on a medical officer of the Philippine Army, your female patient has had multiple episodes of lightheadedness, sweating, fatigue, tremor, and intense hunger. She had one seizure. During two of these episodes, her blood glucose was 40 mg/dL. This patient was desperately trying to get a discharge from the military, and you suspected she was inducing her symptoms by doing which of the following?
a. “Carb loading” before exercise
b. Self-injection of glucagon
c. Self-injection of insulin
d. Taking metformin before exercise

A

d. Taking metformin before exercise

263
Q

The degradation of glycogen normally produces;
a. Neither glucose nor glucose 1-phosphate
b. More glucose 1-phosphate than glucose
c. More glucose than glucose 1-phosphate
d. Equal amounts of glucose and glucose 1-phosphate

A

b. More glucose 1-phosphate than glucose

264
Q

Which one of the following conditions causes hemoglobin to release oxygen more readily?
a. Metabolic alkalosis
b. Increased production of 2,3-biphosphoglycerate
c. Replacement of the beta subunits with gamma subunits
d. Hyperventilation, leading to decreased levels of CO2 in the blood

A

b. Increased production of 2,3-biphosphoglycerate

265
Q

Prevents DNA replication prior to repair of damaged DNA
a. G2/M checkpoint
b. Intra-S checkpoint
c. Spindle checkpoint
d. G1 checkpoint

A

b. Intra-S checkpoint

266
Q

Ferritin is an example of which of the following types of proteins
a. Transport proteins
b. Storage proteins
c. Regulatory proteins
d. Structural proteins

A

b. Storage proteins

267
Q

RB1 gene
a. Caretaker tumor-suppressor gene
b. Gatekeeper tumor-suppressor gene

A

b. Gatekeeper tumor-suppressor gene